Está en la página 1de 42

EXAMEN DIAGNÓSTICO La artrocentesis es un método útil para confirmar la sospecha

diagnóstica y descartar diagnósticos diferenciales mediante la


CASO CLÍNICO 1 observación, análisis y cultivo del líquido.
A 35-year-old woman is brought to the emergency 3. You find purulent fluid. Once you confirm diagnosis,
department complaining of pain and swelling of her right which treatment is indicated?
knee for the past 2 days. She has been taking acetaminophen
for the knee pain during the past 2 days, but the pain is Intravenous antibiotic therapy and hospitalization
worse today. She has not had any trauma or any previous Oral antibiotic therapy and discharge
problems with her joints. She is otherwise healthy and she Surgical debridement and discharge
currently takes an oral contraceptive. She is sexually active Surgical debridement and intravenous antibiotic therapy
and has a 10-year-old son who lives with her. She is allergic
to penicillin. She is a receptionist at a local hotel and she Debido al fluido purulento, es recomendable evacuar la
tells you she must stand often while working. She is 160 cm cavidad articular para evitar daño al cartílago y comenzar
tall and weighs 52 kg; Temperature 37.9°c. The right knee is antibioticoterapia IV. La GPC recomienda artrotomía
erythematous, swollen, and tender; there is pain on evacuadora debe ser considerada siempre que exista
movement. sospecha de infección o sepsis grave, material más denso y
organizado, parámetros inflamatorios muy elevados,
1. What would you expect to find on x-ray? patógenos muy virulentos y articulaciones profundas.
Articular erosion 4. What is the treatment recommended for this
Decreased articular space patient?
Lytic lessions
None Clindamycin
Erythromycin
La paciente no tiene antecedentes traumáticos y tiene Dicloxacylin
solamente dos días de evolución con la sintomatología, por lo Vancomycin
que lo más probable es que no se encuentre ninguna
alteración en una radiografía. Probablemente tenga una Dado que la paciente es alérgica a penicilina, de las opciones
artritis séptica, por lo que, al menos los primeros 7 días, la presentadas de respuesta, la mejor alternativa terapéutica
radiografía no presente ningún cambio. en este momento es clindamicina cada 6 a 8 horas, con una
dosis máxima de 1.8 gr.
2. What is the next step in diagnosis?

Arhrocentesis of the knee


Blood cultures 5. What percentage of patients have medical sequelaes
Complete blood count after an acute episode?
MRI of the knee
20%
40%
50% Hip dislocation
10% Legg Calve Perthers disease

La GPC recomienda el envío a rehabilitación a todos los La enfermedad de Legg-Perthes-Calvé es un proceso


pacientes tras la resolución del cuadro agudo, ya que hasta patológico que afecta al desarrollo de la cadera del niño que
50% de los casos pueden quedar con secuelas. se produce por necrosis aséptica de la cabeza femoral por
alteración idiopática de la circulación. Tomando en cuenta la
CASO CLÍNICO 2 edad y clínica del paciente y la radiografía en donde se puede
apreciar aplanamiento de la epífisis femoral, esta es nuestra
A 7-year-old boy is brought to the pediatrician’s office by his principal sospecha diagnóstica.
mother. She first noticed a limp some days ago. The child
doesn’t refer discomfort and he has no history of trauma or 7. What is the most appropriate treatment for this
recent infection. He is otherwise in good health and has child’s condition?
reached all developmental milestones on time. Vital signs
are within normal limits. Physical examination reveals Chemotherapy alone
limited abduction and internal rotation of his left leg. The Gentle closed reduction
left hip joint shows no evidence of instability or localized Observation and limited weight bearing
pain. Deep tendon reflexes are brisk, pulses are normal, and Surgical intervention with adjuvant chemotherapy
muscle strength testing shows 5/5 strength bilaterally. A
radiograph of the child’s pelvis is shown in the image. El primer paso en el tratamiento para mejorar las condiciones
del paciente y prevenir el colapso óseo en la enfermedad e
Perthes, es limitar la carga de peso en la cadera.

8. Which of the following is the expected progression


of this lesion?

Destruction of local tissue that progress the osteoarthritis


High risk of local recurrence and metastasis
Recurrent local infection
Extension to adjacent hip

Los pacientes con enfermedad de Legg-Perthes-Calvé suelen


progresar a presentación de osteoartitis por destrucción del
tejido de la cadera.

6. What is the diagnosis? CASO CLÍNICO 3

Osteosarcoma A 4,080-g male newborn is delivered at term by a 32-year-old


Hip dysplasia woman, gravida 2, para 1. Apgar scores are 8 and 9 at 1 and
5 minutes, respectively. Examination in the delivery room knee shows swelling, tenderness, warmth, and erythema;
shows fracture of the right clavicle and abscent arm movility. range of motion is limited.

9. What is the diagnosis? 11.Which of the following is the most appropriate next
step in management?
Erb´s palsy
Cerebral palsy Nuclear scan of the right knee
Streeter displasya MRI of the right knee
Amelia Antibiotic therapy
Arthrocentesis
La parálisis braquial obstétrica es la parálisis de los músculos
inervados por raíces nerviosas originadas en el plexo braquial. La artrocentesis es un método útil para confirmar la sospecha
La lesión de las raíces superiores (C5-C6, y puede involucrar diagnóstica y descartar diagnósticos diferenciales mediante la
C7) es denominada parálisis de Erb-Duchenne. Es la observación, análisis y cultivo del líquido.
presentación más frecuente de parálisis braquial obstétrica y
se caracteriza por parálisis del hombro y antebrazo. 12.What would an arthrocentesis show?
Hypertrophic synovial tissue
10.Which of the following is the most likely sequela of The synovial fluid is cloudy
this condition? Symptoms improvement
Lytic lesions
Correction only with casting
Correction only with physical therapy En una artritis séptica el líquido sinovial sería turbio, en lugar
Left-handed dominance de ser claro y translúcido.
Permanent nerve damage
13.Which of the following options is the most
En muchas ocasiones, cuando la lesión nerviosa es severa, la appropriate additional test?
parálisis braquial no muestra mejoría y el daño es Culture for bacteria
permanente. Whole body CT-scan
Polarized light microscopy
CASO CLÍNICO 4 Protein level
A previously healthy 4-year-old girl is brought to the
A pesar de que la sospecha de artitis séptica sea alta, es
physician because of a 2-day history of fever and pain and
importante cultivar el líquido sinovial para encontrar el
swelling of the right knee. She remembers injuring the knee
agente causal específico y su sensibilidad a antibióticos.
while playing soccer last week, but she was able to finish the
game. She has no history of rash or joint pain. Her sister has
14.Which of the following is the most likely causal
inflammatory bowel disease. The patient's temperature is
organism?
39°c (102.2°f), blood pressure is 110/80 mm hg, pulse is 95/
Haemophilus influenzae type b
min, and respirations are 20/min. Examination of the right
Staphylococcus aureus
Streptococcus pneumoniae Prescribe acetaminophen and muscle relaxant
Streptococcus pyogenes (group a) Ask for x-rays
Prescribe NSAIDs and opioids
Staphylococcus aureus es el patógeno más frecuentemente
vinculado a artitis séptica, lo cual es muy importante tener El paracetamol se considera el fármaco de primera línea,
en cuenta al momento de establecer antibioticoterapia indicado en lumbalgia aguda. Puede prescribirse relajante
empírica. muscular si el paciente se refiere con espasmo muscular.

15.Which of the following studies is useful in this 17.In addition to analgesia, which of the following is
disease? the most appropriate next step in management?
X-Ray Full bed rest
Ultrasaound X-rays of the spine
Computed tomography Regular activity
All of above. MRI of the spine

La radiografía simple debe ser el primer estudio de imagen La recomendación al paciente de continuar con su actividad
realizado, aunque frecuentemente se encuentras pocos regular, ha demostrado que ayuda a mejorar la morbilidad en
hallazgos en artritis séptica. El ultrasonido se encuentra la lumbalgia y hace más fácil la recuperación. Sin embargo,
recomendado para búsqueda intencionada de colecciones y se le debe dar orientación al paciente sobre cuestiones
determinar las características de las mismas, realizándose de posturales, y la manera de realizar sus actividades.
forma comparativa y tomar medidas de la cápsula. La
tomografía debe realizarse en casos de evolución subaguda o 18.In case of radiculopathy which is the most affected
crónica al no contar con RM. La resonancia magnética deberá intervertebral disc space?
realizarse de primera intención en caso de tener evolución L2 – L3
tórpida, alta sospecha de complicaciones y falta de repuesta. L3 – L4
L4 – L5
CLINICAL CASE 5 L5 – S1

A 37-year-old man comes to the physician because of non- Estadísicamente, el nivel más afectado en caso de
irradiated low back pain for 3 days. The pain began after he radiculopatía es L4-L5, seguido por L5-S1.
worked in his yard. He has not had any change in bowel
movements or urination. He had one similar episode 3 years 19.How do you explore L5 sensitivity?
ago that resolved spontaneously. Vital signs are within Lateral leg
normal limits. Examination of the back shows bilateral Posterior leg
paravertebral muscle spasm. Range of motion is limited by Around the knee
pain. Straight-leg raising is negative. Above the knee

16.What would you do next?


Ask for previous neoplastic disease
La exploración de carácter sensitivo de L5 incluye la cara 22. Which is the principal risk factor in the
externa de la pierna, la mitad interna del dorso del pie development of symptoms?
incluyendo el dedo mayor. History of trauma
Hands physical activity
20.How do you explore L5 motion function? Oral contraceptive therapy
Ankle plantar flexors Obesity
Hip flexors
Knee extensors En general, las afecciones inflamatorias, se pueden ver
Long toe extensors influenciadas en su desarrollo por micro trauma durante la
actividad física.
Para valorar L5 se pide al paciente que camine de puntillas,
así como la valoración de la extensión. Debe sospecharse 23.Which of the following is not indicated treatment
lesión si la respuesta es oponerse al movimiento de flexión for this disease?
del pie. Antidepressant therapy
NSAIDs therapy
CLINICAL CASE 6 Rehabilitation therapy
A 25-year-old woman comes to the physician because of a 2- Analgesic therapy
month history of numbness in her right hand. During this
period, she has had tingling in the right ring and small El reposo, rehabilitación y medidas analgésicas están
fingers most of the time. She has no history of serious illness indicadas en este caso. La terapia con antidepresivos, no
and takes no medications. She is employed as a cashier and juega ningún papel.
uses a computer at home. She played as a pitcher in a
softball league for 5 years until she stopped 2 years ago. 24.What’s the name of the sign that produces a jolt of
Vital signs are within normal limits. Examination shows full severe pain or tingling when percussing over the
muscle strength. Sensation to pinprick and light touch is nerve?
decreased over the medial half of the right ring finger and Phallen
the entire small finger. Filkenstein
Hoffman
21.The most likely cause of these findings is Tinel
entrapment of which of the following on the right?
Brachial plexus at the axilla El signo de Tinel es un signo clínico utilizado para detectar la
Median nerve at the wrist irritación o inflamación de un nervio. Es positivo cuando, al
Ulnar nerve at the elbow percutir sobre el trayecto de un nervio, aparecen parestesisas
Radial nerve at the forearm y calambres en el territorio inervado por este.

El cuadro concuerda con un atrapamiento del nervio cubital 25.Which entrapment would cause a fick sign?
(síndrome del túnel cubital), presentándose con parestesias Brachial plexus
de la mano y los dedos. El lugar más común de compresión Medial nerve
del nervio es detrás de la parte interna del codo. Radial nerve
Ulnar nerve Osteoblastoma

“Fick sign” o signo de la sacudida se da en fases iniciales de La principal sospecha por el contexto clínico y la imagen
compresión del nervio medial. La persona tiene la necesidad encontrada es osteosarcoma. El patrón permeativo en la
de sacudir la mano para reducir la sintomatología nocturna. cortical, la reacción perióstica y aumento de volumen de
partes blandas sugieren malignidad.

CASO CLÍNICO 7 27.Ante tal sospecha, ¿cuál es su siguiente conducta?


Referencia urgente a 2do o 3er nivel de atención
Masculino de 11 años que es traído por sus padres porque Iniciar antinflamatorio y terapia física
desde hace 10 días presenta febrícula diaria de predominio Realizar artrocentesis
vespertino. En los últimos 2 días la fiebre ha ido aumentando Agendar cita control en 1 mes con nuevas radiografías
(hasta 38.5 °C). Tiene poco apetito y ha perdido peso (2 kg).
Hace 3 semanas le dieron una patada en la rodilla izquierda. Las guías de práctica clínica indican que ante la sospecha de
Inicialmente sólo tenía dolor local, posteriormente notaron malignidad siempre se debe referir al centro dónde se llevará
inflamación que fue en aumento hasta impedirle caminar. el tratamiento definitivo.
Sus estudios de laboratorio reportan: PCR 75 mg/L, sin otras
alteraciones significativas. Se realiza radiología simple: 28.¿Cuál de los siguientes estudios no tiene indicación
en el protocolo de estudio de este paciente?
26.¿Cuál de los siguientes es el diagnóstico más Resonancia magnética del muslo y rodilla
probable? Tomografía Axial de rodilla y tórax
Osteomielitis Análisis de dinámico de la marcha
Granuloma eosinófilo PFH, Perfil óseo, BH
Osteosarcoma
Los estudios locales y de extensión de la lesión son el
complemento de estudio del paciente oncológico. Es
importante realizar estudios de laboratorio e imagen
complementarios. El análisis dinámico de la marcha, no tiene
un papel importante en este caso.

29.¿Cuál es el tratamiento indicado de esta lesión tras


confirmar diagnóstico?
Terapia física y fortalecimiento muscular
Resección intralesional e injerto óseo
Desbridamiento y antibioticoterapia
Quimioterapia y resección amplia

En este paciente con osteosarcoma, el tratamiento más


adecuado sería neoadyuvancia con quimioterapia para control
local y de micrometástasis tras resección con márgenes 32.¿A qué se debe el derrame articular en este
amplios. padecimiento?
Sinovitis
30.¿Cuál es la principal complicación tras el Hemartrosis
tratamiento quirúrgico? Reacción inflamatoria
Recidiva a distancia Reacción mecánica
Infección
Consolidación viciosa Dado el mecanismo de la lesión, la sospecha es de una
Todas las anteriores hemartrosis, muy probablemente asociada a la ruptura a la
arteria del ligamento cruzado anterior.
Debido al estado inmunológico y la magnitud del
procedimiento de resección la infección es la complicación 33.¿Qué signo clínico reafirmará la sospecha
más común tras un procedimiento quirúrgico de resección de diagnóstica?
osteosarcoma, seguido de la recidiva local. Mc Murray
Steinman
Ficat
CASO CLÍNICO 8 Lachman

Masculino de 23 años de edad que al realizar un salto Las principales maniobras para la valoración diagnóstica de
jugando al baloncesto, cae sobre su extremidad inferior lesión del ligamento cruzado anterior son: Lachman,
derecha con la rodilla en hiperextensión aplicando un giro maniobra de cajón anterior y maniobra pivot shift.
brusco a su rodilla mientras que mantiene el pie fijo en el
suelo. El paciente, percibe un chasquido y dolor agudo en su 34.De los siguientes ¿qué signo clínico puede encontrar
rodilla, no pudiendo continuar jugando. Nota sensación de positivo?
inestabilidad al realizar el apoyo de dicha extremidad. En la Pivot Shift
exploración clínica se aprecia derrame articular intenso. La Bostezo lateral
movilidad de la rodilla está libre. Appley
Ninguno de los anteriores
31.¿Cuál es la sospecha diagnóstica?
Rotura en asa de cubo del menisco interno La maniobra de Pivot Shift se explora con la rodilla en
Rotura aislada del ligamento colateral lateral completa extensión. Se aplica valgo y una rotación medial, se
Fractura por arrancamiento de la espina tibial anterior flexiona la rodilla y, posteriormente, se lleva a extensión
Lesión del ligamento cruzado anterior completa de nuevo. En caso de un ligamento cruzado anterior
roto, se puede percibir un chasquido característico.
Por el mecanismo de la lesión y la presentación clínica
actual, se sospecha de ruptura del ligamento cruzado 35.¿En qué consiste en tratamiento inmediato de ésta
anterior. Debe sospecharse esta entidad en jóvenes lesión?
deportistas con mecanismos rotacionales y posterior Analgésico, inmovilización y medios físicos
inestabilidad. Artrocentesis, inmovilización y lavado articular
Analgésico, terapia física y ejercicio de impacto El síndrome de Legg-Calvé-Perthes, es una enfermedad
Todas las anteriores infantil que cursa con necrosis de la cabeza del fémur. Por el
contexto clínico, y los hallazgos radiográficos de cambios en
La primera línea de tratamiento de una lesión del ligamento el tamaño, lucidez y aplanamiento de la epífisis femoral
cruzado anterior requiere control de dolor, estabilización de sugieren cambios vasculares.
la articulación y disminución de edema por medios físicos.
37.¿Cuál es el tratamiento inmediato para este
padecimiento?
CASO CLÍNICO 9 Quimioterapia
Reducción abierta
Niño de 6 años que presenta dolor inguinal derecho y cojera, Modificar actividad física, limitar carga
de presentación insidiosa y de un mes de evolución. No Resección amplia y quimioterapia
refiere fiebre, ni traumatismos o infecciones previas. En la
exploración se aprecia limitación a la abducción y rotación El primer paso en el tratamiento para mejorar las condiciones
interna de la cadera derecha, marcha con discreta cojera sin del paciente y prevenir el colapso óseo en la enfermedad e
dolor. Resto de exploración sin hallazgos patológicos. Se Perthes, es limitar la carga de peso en la cadera.
realiza la radiografía.
38.¿Cuál de los siguientes es un factor de riesgo para el
desarrollo de la enfermedad?
Historia de trombosis
Infecciones previas
Anomalías en la coagulación
A y C son correctas

El origen de la enfermedad de Perthes no está claro. Algunas


teorías la relacionan con haber padecido previamente
sinovitis. También hay teorías que apuntan a los fenómenos
que ocasionan cambios vasculares en la coagulación.

39.¿Cómo esperaría encontrar los reactantes de fase


aguda al momento del diagnóstico?
PCR elevada y VSG normal
PCR normal y VSG normal
PCR normal y VSG elevada
36.¿Cuál es su diagnóstico?
PCR elevada y VSG elevada
Enfermedad de Perthes
Sinovitis transitoria de caderas
Dado que en la enfermedad de Perthes no hay un componente
Artritis séptica de cadera
infeccioso o propiamente inflamatorio, sino que es de origen
Osteosarcoma de fémur proximal
necrótico, los reactantes de fase aguda, habitualmente son 42.C o r r e s p o n d i e n d o a l a c l a s i f i c a c i ó n d e l a
normales. enfermedad:
Piógena
40.¿Cuál es la principal complicación a largo plazo de Inespecífica
esta enfermedad? Crónica estable
Recidiva local Velonodular
Infección recurrente
Inestabilidad de cadera Por la duración de los síntomas, la epifisiolisis femoral
Artrosis proximal se clasifica como predeslizamiento, aguda, aguda
sobre crónica o crónica. Debido a que la duración en este
La deformidad residual tras la consolidación en la paciente es mayor a 3 meses y que puede realizar marcha, se
enfermedad de Perthes lleva a artrosis a mediano y largo considera crónica estable
plazo, siendo esta la complicación más frecuente en esta
entidad. 43.¿Cuál es el signo diagnóstico esperado en la
radiografía?
CASO CLÍNICO 10 Disminución de índice cefálico
Aumento de índice acetabular
Masculino de 14 años de edad, con 158 cm de altura y 76 kg Trethowan
de peso, acude a tu consulta por presentar desde hace 4 Erosión acetabular
meses, sin antecedente traumático previo, dolor en región
inguinal derecha irradiado a rodilla ipsilateral. En la Al trazar una línea por el lado superior del cuello femoral
exploración clínica, claudicación a expensas de miembro cruza la porción superior de la epífisis en la cadera normal
pélvico derecho con tendencia a la rotación externa a la (línea de Klein). En el deslizamiento, esta línea no pasa por la
flexión pasiva y limitación de la rotación interna de cadera. epífisis o la cruza en menor grado que en la cadera sano. A
Signos vitales en rangos normales. este signo se le conoce como signo de Trethowan.

41.El diagnóstico de sospecha será: 44.El tratamiento profiláctico en este paciente


Epifisiolisis femoral proximal correspondería a:
Enfermedad de Perthes Aplicación intraarticular de itrio
Artritis séptica de cadera Colocación de aparato de descarga
Sinovitis transitoria de cadera Antibioticoterapia intravenosa
Fijación de cadera contralateral
En pacientes masculinos obesos durante la pubertad, que
presentan rotación de cadera limitada se debe sospechar En presencia de factores de riesgo para epifisiolisis femoral
deslizamiento epifisiario femoral respecto al cuello femoral proximal, se debe fijar la cadera contralateral aún en
(epifisiolisis femoral proximal) como primera posibilidad ausencia de síntomas o cambios radiológicos.
diagnóstica.
45.La principal complicación a largo plazo de este caso
sería:
Artrosis La clasificación de Gustillo y Anderson clasifica el grado según
Consolidación viciosa la dimensión. grado I= menor a 1cm, grado II: entre y 10cm,
Infecciones repetitivas grado III: >10cm. El caso clínico menciona que este paciente
Inestabilidad articular tiene un grado III.

Los cambios morfológicos o la necrosis avascular que ocurren 48.¿Cuál es el principal riesgo de complicación de no
en casos de deformidades severas, conducen a artrosis a recibir un tratamiento oportuno?
mediano y largo plazo. Retardo de consolidación
Infección
Consolidación viciosa
CASO CLÍNICO 11 Deformidad Angular

Paciente de 30 años de edad quien tras sufrir un accidente de La infección local por colonización bacteriana en el tejido
moto presenta una fractura abierta grado III de tibia y necrótico incrementa con el tiempo transcurrido a partir de
peroné sin afectación neurovascular. la inoculación, siendo esta la principal complicación.
46.¿Cuál sería la conducta a seguir?
Limpieza de la herida y osteosíntesis inmediata con placa y 49.El trazo de fractura a 3 cm de la cabeza de peroné
tornillos obliga a descartar qué tipo de lesión:
Limpieza de la herida, estabilización provisional de la Inestabilidad de rodilla
fractura con tracción continua y yeso tras obtenerse la Lesión de nervio ciático poplíteo externo
curación de la herida cutánea Lesión de músculo peroneo largo
Limpieza de la herida, estabilización provisional con tracción Lesión de músculo peroneo corto
y posterior colocación de aparato corto de yeso
Limpieza de la herida y estabilización de la fractura con Por la proximidad anatómica con el sitio de lesión, es
fijador externo importante verificar la integridad y función del nervio ciático
poplíteo externo.
La descontaminación de los tejidos a la brevedad y
estabilización del segmento son el control de daños inicial 50.Clínicamente, ¿qué esperaría encontrar para
antes del tratamiento definitivo que debe seguir a la corroborar lesión del nervio ciático poplíteo
antibioticoterapia. externo?
Ausencia de dorsiflexión de tobillo
47.De acuerdo a la clasificación de Gustilo-Anderson, Bostezo lateral positivo
¿qué dimensión tiene la herida del paciente? Bostezo medial positivo
De 1 a 3 cm Cajón anterior en tobillo
De 1 a 5 cm
De 5 a 10 cm El nervio ciático poplíteo externo (L4-S2) proporciona los
Mayor de 10 cm nervios sensitivos sural lateral y cutáneo lateral de la
pantorrilla. En caso de lesión de este, encontraríamos
alteraciones en la dorsiflexión y alteraciones de sensibilidad opciones presentadas forman parte de la articulación del
en dorso del pie. hombro, excepto la clavículo – torácica.

CASO CLINICO 1 3. ¿Qué músculos conforman el manguito rotador?


Supraespinoso, infraespinoso, redondo mayor y redondo
Femenino de 47 años que acude a consulta refiriendo dolor mayor
en hombro derecho de 1 semana de evolución irradiado a Supraespinoso, infraespinoso, subescapular y bíceps
cara lateral de brazo que relaciona posterior a tender ropa e Supraespinoso, infraespinoso, subescapular y redondo menor
incrementa por las noches. Refiere actualmente ser maestra Supraespinoso, redondo menor, subescapular y deltoides
jubilada, ser madre de 3 hijos en edad escolar y adolescente
y no realizar ningún tipo de actividad física recreativa de El manguito de los rotadores es un es el conjunto de músculos
manera habitual. Niega pérdida de peso o síntomas y tendones que dan estabilidad al hombro, conectando la
constitucionales. A la EF constitución fina con leve hipotrofia escápula con la cabeza del húmero. Los músculos que incluye
de Deltoides con arcos de movilidad de hombro completos y son: Supraespinoso, infraespinoso, subescapular y redondo
signos vitales en parámetros normales. menor.

1. De acuerdo a su sospecha diagnóstica ¿Qué 4. ¿Cuál de los siguientes músculos no tiene relación
antecedente referido es de mayor importancia? con el canal bicipital?
Antecedente laboral como maestra Porción corta del bíceps
Sedentarismo Porción larga del bíceps
Multiparidad Pectoral mayor
Síntomas constitucionales Dorsal ancho
La porción corta del bíceps braquial, se origina en la apófisis
En las enfermedades inflamatorias degenerativas los coracoides por un tendón común con el coracobraquial. La
antecedentes laborales son el principal factor predisponente, porción larga, se origina en tuberosidad supraglenoidea de la
por lo cual, el antecedente de haber trabajado como escápula (omóplato) y desciende por la corredera bicipital del
maestra, resulta importante en este caso. húmero. Ambos cuerpos musculares se reúnen, e insertan
inferiormente mediante un tendón común, en la tuberosidad
2. ¿Cuál de las siguientes no forma parte de la bicipital del radio.
articulación del hombro?
Escápulo-torácica 5. Dados los hallazgos clínicos ¿Cuál es la anomalía
Gleno-humeral anatómica más probable?
Acromio-clavicular Ruptura parcial de subescapular
Claviculo-torácica Ruptura parcial de supraespinoso
Ruptura parcial de infraespinoso
La articulación del hombro incluye el conjunto de Ninguna de las anteriores
articulaciones involucradas en el movimiento del brazo
proximal desde el esternón hasta la escápula, todas con
mecanismos bien definidos y mecánica sinérgica. Todas las
A la exploración física, los arcos de movilidad se encuentran que inerva la parte lateral de la palma y la rama cutánea
completos, lo cual sugiere que las estructuras anatómicas de digital, que inerva los tres dedos laterales y medio en la
la articulación se encuentran íntegras. superficie anterior de la mano.

8. ¿Qué dedos esperaría encontrar con hipoestesia en


CASO CLINICO 2 caso de compresión de N. Cubital?
Dorso de 1er y 2do dedos
Femenino de 53 años de edad, secretaria de escuela 4to y mitad cubital de 5to dedos
secundaria que acude a consulta refiriendo desde hace 2 Cara anterior de 1ro, segundo y 3er dedo
meses debilidad en mano derecha acompañada de Dorso de 2do, 3ero y mitad cubital de 4to dedo
adormecimiento ocasional. Refiere dejar caer objetos por
falta de fuerza en 2 ocasiones. Niega antecedentes
traumáticos, esguinces o fracturas previas. Cuenta con RX AP El nervio cubital cubre tanto funciones motoras como
y Lat de muñeca y mano de hace un mes en las cuales no se sensoriales. Inerva y da función motora a diferentes músculos
evidencia lesión ósea. EF con peso de 87 kg y talla de 1.57m, del antebrazo y mano. En cuanto a la función sensitiva,
signos vitales normales. proporciona inervación y percepción en los costados palmar y
dorsal del quinto y cuarto dedos.
6. De acuerdo a la descripción del padecimiento ¿Cuál
es la principal sospecha diagnóstica?
Compresión radicular cervical 9. ¿Qué elemento no forma parte del túnel carpiano?
Compresión nerviosa periférica Nervio cubital
Enfermedad reumatológica Nervio mediano
Fractura por estrés Flexor común superficial de los dedos
Flexor común profundo de los dedos
Por la ocupación y el cuadro de la paciente, se sospecha de
síndrome de túnel del carpo, que es la principal causa de El túnel del carpo incluye al ligamento transverso del carpo,
dolor en muñeca. Se presenta con datos de compresión el nervio mediano, los tendones flexores, las vainas de los
nerviosa por atrapamiento del nervio mediano. tendones y los huesos del carpo. El nervio cubital no es parte
de esta estructura.
7. ¿Qué dedos esperaría encontrar con hipoestesia en
caso de compresión de N. Mediano? 10.¿Cómo está conformada la hilera proximal de los
Dorso de 1er y 2do dedos huesos del carpo?
4to y mitad cubital de 5to dedos Trapecio, Trapezoide, Grande y Ganchoso
Cara anterior de 1ro, segundo y 3er dedo Trapecio, Trapezoide, Piramidal y Pisiforme
Dorso de 2do , 3ero y mitad cubital de 4to dedo Escafoides, Piramidal, Grande y Ganchoso
Escafoides, Semilunar, Piramidal y Pisiforme
El nervio mediano es un nervio mixto. Dentro de sus funciones
sensoriales en la mano, da lugar a la rama cutánea palmar,
Los huesos del carpo son: fila proximal: escafoides, El signo de Lassegue es un dato clínico de neurotensión que
semilunar, piramidal y pisiforme; fila distal: trapecio, suele ser positivo en pacientes con lumbalgia aguda y
trapezoide, hueso grande y hueso ganchoso. contractura paravertebral severa.

CASO CLINICO 3 13.¿Cuál es el fármaco de primera línea en el manejo


de dolor lumbar agudo?
Masculino de 27 años de edad, sin antecedentes oncológicos Tramadol
familiares o personales, Acude al servicio de urgencias en Ibuprofeno
silla de ruedas refiriendo un par de horas de evolución de Dextropopoxifeno
dolor lumbar intenso irradiado a miembro pélvico izquierdo Paracetamol
que inició tras levantar rejas de alimentos y posteriormente
le impide la incorporación y marcha. A la EF con contractura El paracetamol en dosis menor a 2 gramos por día es el
muscular paravertebral dolorosa, con fuerza muscular 5/5 medicamento de elección de primera línea para dolor lumbar
bilateral global, sensibilidad de L1 a S1 2/2 bilateral global. agudo.
Signos vitales en parámetros normales, no se palpan ganglios
linfáticos. 14.¿A qué nivel medular corresponde la integración del
reflejo Aquíleo?
11.¿Qué diagnóstico es el más probable ante este L4
cuadro? L5
Fractura de T12 S1
Lumbalgia mecánica con radiculopatía S2
Lumbalgia mecánica sin radiculopatía
Metástasis ósea en columna Ante la percusión del tendón de Aquiles, se contrae el
músculo tríceps sural, que produce la extensión plantar del
La lumbalgia mecánica aguda con radiculopatía comúnmente pie. Con esta acción se explora así la raíz S1, en donde se
se presenta tras esfuerzos en extensión de la columna dorso- integra el reflejo.
lumbar en pacientes jóvenes, como es el caso de este
paciente. 15.¿En qué consiste la respuesta refleja del nivel L3?
Flexión plantar del pie
12.¿Qué signo clínico es comunmente positivo en estos Extensión del primer dedo y flexión del resto
pacientes? Flexión de cadera
Babinsky Extensión de rodilla
Thompson
Patrick El reflejo rotuliano es la contracción involuntaria del músculo
Lassegue cuádriceps femoral por la flexión del tendón rotuliano. Es un
reflejo de tipo miotático o de estiramiento. Se integra a nivel
de L3.
CASO CLINICO 4 Contractura de tendón de Aquiles
Contractura de aductores
Masculino de 57 años de edad con antecedente de gota de 30 Hipotrofia de cuádriceps
años de evolución en tratamiento con Alopurinol y dieta. Hipotrofia de gemelos
Refiere gonalgia derecha de 1 año de evolución que ha
incrementado en el último més de predominio tras la La disminución del trofismo muscular aparece como
bipedestación o marcha prolongadas. EF con marcha consecuencia de la reducción en el uso del músculo como
claudicnate dercha con alineación en varo de ambas rodillas mecanismo preventivo de dolor. En este caso, esperaríamos
con aumento de volumen de rodilla derecha, flexión de 100 encontrar esta hipertrofia en el cuadríceps derecho.
grados, extensión de cero. Niega pérdida de peso o fiebre. TA
de 120/90, FC 83, Temp 36.5°C, 1.75m, 97kg. 19.Además de evitar actividad de impacto, mejorar el
tono y movilidad articular ¿Qué factor se debe
16.Ante los datos clínicos del paciente ¿Cuál es su corregir como medida terapéutica?
principal sospecha diagnóstica? Obesidad
Gonartrosis derecha Gota
Crisis aguda de gota Marcha claudicante
Lesión de menisco medial Movilidad (flexión de 100 grados)
Lesión de menisco lateral
El control de peso es una medida que ha demostrado
La osteoartrosis es una enfermedad degeneratva disminuir el estrés en carga y con esto aliviar la intensidad
multifactorial que normalmente afecta a individuos en la del dolor.
quinta década de la vida en adelante. El tipo del dolor que
tiene el paciente es progresivo, no tiene datos de un ataque 20.De acuerdo a la clasificación de Kellgren y Laurence
agudo, lo que apoya este diagnóstico. ¿En qué estadio deben estar los pacientes ideales
para tratamiento quirúrgico?
17.¿Cuál es el factor de riesgo para el desarrollo de 0 y 1
esta enfermedad? 1 y 2
Obesidad 2 y 3
Alineación en varo 3 y 4
Gota
Todas las anteriores La clasificación de Kellgren y Laurence categoriza en 4 grados
la osteoatrosis. Grado 0: normal. Grado 1: dudoso.- Grado 2:
La osteoartorisis tiene un origen multifactorial. Todos los leve- Grado 3: moderado. Grado 4: grave. El tratamiento
datos presentados en las opciones pueden contribuir al quirúrgico se recomienda para estadios de daño articular 3 y
desarrollo de esta entidad. 4.

18.De acuerdo con el tiempo de evolución y los


hallazgos clínicos ¿Qué cambios musculares
esperaría encontrar?
CASO CLINICO 5 Articulación cuneo-metatarsiana

Femenino de 31 años de edad, abogada activa, con La lesión consiste en la desviación en valgo del primer ortejo
antecedente de apendicectomía en la infancia que acude a con una desviación en varo del primer metatarsiano,
consulta por presentar dolor moderado en primer dedo de afectando la articulación metatarso falángica.
ambos pies con deformidad progresiva del mismo. EF con
deformidad en valgo de articulación metatarso falángica con 24.¿Cuál es la principal complicación que se presenta
eritema en región lateral de la misma, movilidad articular después de tratamiento quirúrgico?
completa, con dolor mínimo. Signos vitales en parámetros Sobrecorrección
normales. Acortamiento
Persistencia de dolor
21.¿Qué estudio de imagen debe solicitar para Infección
complementar diagnóstico?
RX DP y LAT bilateral de pies La persistencia o aparición de dolor es la principal
RX DP y LAT de tobillos complicación tras este procedimiento. Se ha observado sobre
RX DP y LAT de pies con apoyo todo en aquellos tratamientos que se realizan por indicación
RX DP y LAT de tobillos con apoyo estética.

Para la valoración de hallux valgus, como en el caso de esta 25.¿Cuál es la deformidad más comunmente asociada?
paciente se deben solicitar 2 proyecciones en carga de los Dedos en garra
pies, dorsoplantar y lateral. Hallux rigidus
Segundo dedo en martillo
22.¿Cuál es principal factor desencadenante que se Subluxación metatarso falángica
debe corregir?
Desbalance muscular El hallux valgus frecuentemente se encuentra asociado el
Laxitud ligamentaria segundo dedo en martillo debido a la superposición sobre el
Inestabilidad articular hallux desviado.
Calzado estrecho
CASO CLINICO 6
El uso de calzado adecuado (evitar calzado estrecho) es el
principal factor corregible en hallux valgus, que otorga Masculino de 21 años de edad que es llevado al servicio de
beneficios a corto y largo plazo en la sintomatología y urgencias por presentar dolor intenso y derrame articular
progresión de la lesión. después de recibir una barrida sobre la cara lateral de la
rodilla derecha al jugar soccer. Además del dolor intenso, la
23.¿Sitio anatómico en el que se lleva a cabo la lesión le condicionó incapacidad funcional que le impidió
deformidad? terminar el partido.
Articulación interfalángica distal
Articulación interfalángica proximal 26.¿A qué se debe el derrame articular en este
Articulación metatarso-falángica padecimiento?
Sinovitis Tomando en cuenta el mecanismo de lesión que tuvo este
Hemartrosis paciente, con el valgo forzado una lesión de ligamento
Inflamatoria colateral medial arroja bostezo medial positivo.
Mecánica
30.¿En qué consiste en tratamiento inmediato de ésta
Dado el mecanismo de la lesión, la sospecha es de una lesión?
hemartrosis, muy probablemente asociada a la ruptura del Analgésico, inmovilización y medios físicos
ligamento cruzado anterior. Artroscentesis, inmovilización y lavado articular
Analgésico, terapia física y ejercicio de impacto
27.¿Cuál es el diagnóstico más probable con esta Todas las anteriores
clínica?
Lesión de menisco medial La primera línea de tratamiento de una lesión del ligamento
Lesión de menisco lateral cruzado anterior requiere control de dolor, estabilización de
Lesión de retináculo medial la articulación y disminución de edema por medios físicos.
Lesión de ligamento cruzado anterior

Por el mecanismo de la lesión, lo más probable es que se Displasia en el desarrollo de cadera / Pie equino varo /
haya producido una lesión del ligamento cruzado anterior por Artritis séptica / Lesiones plexo braquial / Fracturas
mecanismo de valgo forzado, provocando secundariamente incompletas
una hemartrosis.
CASO CLÍNICO 1
28.¿Qué signo clínico reafirmará la sospecha
diagnóstica? Femenino de 2 semanas de edad, es llevada a consulta de
Mc Murray control de niño sano. A la exploración física se advierte una
Appley asimetría de los pliegues de la región posterior del muslo.
Lachman
Steinman 1. ¿Qué antecedente de relevancia interrogaría a la
madre?
Las principales maniobras para la valoración diagnóstica de Historia familiar de neoplasias
lesión del ligamento cruzado anterior son: Lachman, Tabaquismo durante el embarazo
maniobra de cajón anterior y maniobra pivot shift. Historia familiar de displasia de cadera
Historia familiar de poliomielitis
29.¿Qué otro signo clínico puede encontrar positivo
acorde al mecanismo de lesión? El antecedente familiar junto con el sexo femenino son los
Bostezo medial principales factores de riesgo para la displasia congénita de
Bostezo lateral cadera.
Escape
Ninguno de los anteriores
2. ¿Cuáles son las principales maniobras diagnósticas Referencia a ortopedia en 2do nivel de atención
de exploración que realizaría? Conducta expectante de acuerdo a síntomas
Galeazzi y Trendelemburg Solicitar estudios de laboratorio
Ortolani y Barlow
Pistón y Ortolani Tras el diagnóstico de la displasia congénita de cadera debe
Barlow y Galeazzi ser indicado y vigilado por el especialista capacitado.

Ortolani (cadera luxada) y Barlow (cadera luxable) son las


maniobras de exploración positivas de mayor utilidad en el CASO CLÍNICO 2
diagnóstico de DDC hasta los 3 meses de edad.
Masculino de 5 días de nacido, de embarazo sin control
3. De acuerdo a la edad de la paciente ¿cuál es el prenatal, producto de término de parto vaginal atendido en
e s t u d i o d e i m a g e n r e c o m e n d a d o p a ra l a la vivienda de paciente por partera, es llevado a consulta por
confirmación del diagnóstico? la madre, por presentar deformidad en un pie que le impide
Artrografía colocarle calzado.
Tomografía Axial Computada
Radiografía simple AP de Pelvis 6. De acuerdo a la descripción del padecimiento, ¿cuál
Ultrasonido es la principal sospecha diagnóstica?
Pie equino varo aducto congénito
Desde el nacimiento y hasta los 3 meses de edad el Pie plano valgo
ultrasonido es el método de imagen de elección para Parálisis cerebral infantil
diagnóstico de displasia congénita de cadera. Pie cavo

4. ¿Qué parámetros se miden en el estudio de imagen El pie equino varo aducto congénito es la deformidad del pie
de elección? más común afectando 2 de cada 1000 nacidos vivos. A pesar
Ángulos Alfa y Beta de que el caso aporta pocos datos, por la descripción y por
Tamaño y vasculatura de la cabeza femoral epidemiología es la opción diagnóstica más probable.
Líneas de Perkings y Hilgenreiner
Unidades Hounsfield en la cabeza femoral 7. ¿En qué relación de género se encuentra
habitualmente ésta patología?
El ultrasonido es el método de imagen de elección para Hombres 1:1 Mujeres
diagnóstico de displasia congénita de cadera en niños de 0 a 3 Mujeres 2:1 Hombres
meses. Los parámetros objetivos de medición para el Hombres 2:1 Mujeres
diagnóstico de DDC son los ángulos Alfa y Beta. Mujeres 3:1 Hombres

5. Tras la confirmación diagnóstica, ¿cuál sería su Estadísticamente el pie equino varo aducto congénito afecta
conducta a seguir? principalmente hombres, hasta en el doble de ocasiones que
Indicación de uso de doble pañal a las mujeres.
Agendar cita de vigilancia en 2 semanas
8. ¿Cuál es el principal factor que modifica el 11.¿Qué antecedente patológico de importancia debe
pronóstico de ésta enfermedad? interrogar?
Si se presenta uni o bilateral Fracturas previas
La atención del parto en un centro hospitalario Enfermedades exantemáticas
El tiempo de inicio del tratamiento Cuadros infecciosos previos
La edad gestacional al nacimiento Esguinces o lesiones deportivas

En el pie equino varo aducto congénito, a menor demora en Tras la sospecha de artritis séptica de debe interrogar sobre
el tiempo de inicio de tratamiento mejor es el pronóstico cuadros infecciosos recientes, predominantemente de la vía
para la función. aérea.

9. ¿Cuál sería el tratamiento de elección indicado en 12.¿Qué estudio paraclínico solicitaría de manera
éste caso? inmediata?
Colocación de yesos seriados Rx AP de pelvis y en posición de rana
Uso de férulas sólo por la noche Gammagrafía de todo el esqueleto
Uso de férulas por 2 semanas Laboratorio (BH, QS, VSG, PCR, PFH)
Colocación de yeso por 3 semanas Cultivos de exudado faríngeo

El método Ponsetti es el tratamiento de elección para el El examen completo refleja el estado general del paciente y
tratamiento de PEVAC y consiste en la aplicación de yesos evidencia los parámetros infecciosos y reactantes de fase
seriados por 3 meses. aguda elevados. Aunque ninguno de estos es específico para
confirmar la sospecha de artritis séptica, son útiles.
10.¿Cuál es la principal complicación que presenta esta
patología? 13.¿Qué procedimiento realizaría a continuación en el
Recidiva de deformidad servicio de urgencias?
Retracción cutánea Artrocentesis
Deficiencia muscular Colocación de férula posterior
Incongruencia ósea Colocación de aparato de yeso tipo espica
Biopsia por punción
La persistencia o recidiva de la deformidad es la principal
complicación, ya sea completa o parcial, es indicativa de La artrocentesis es un procedimiento diagnóstico poco
manejo quirúrgico. invasivo y fácil de realizar que confirma la sospecha
diagnóstica, para después recurrir la inmovilización que
CASO CLÍNICO 3 mejora la sintomatología dolorosa hasta el tratamiento
definitivo.
Masculino de 2 años 2 meses de edad. Presenta claudicación a
la marcha y dolor en ingle izquierda, tiene limitación a la 14.¿Qué hallazgos esperaría encontrar en el estudio
movilización de cadera, sin hematomas o aumento de antes solicitado?
volumen y temperatura de 37.7 °C desde hace 2 días. Pérdida de la continuidad ósea en fémur proximal
Aumento de actividad metabólica en cadera izquierda y 17.¿Qué mecanismo interrogaría a la madre con esta
ambas rodillas sospecha?
Leucocitosis, trombocitosis y aumento de VSG y PCR Tracción axial desde mano o muñeca
Desarrollo de cocos Gram positivos Varo forzado de codo
Carga axial en mano con muñeca en extensión
Aunque la leucocitosis no es un parámetro obligado, la Extensión forzada de codo
elevación de los reactantes de fase aguda suele estar
presente en el 90% de los casos de artritis séptica. La subluxación de la cabeza del radio (llamado comúnmente
codo de niñera) se produce por la tracción de la articulación
15.¿Qué tratamiento definitivo se debe de instaurar? del codo desde el extremo distal del miembro superior (mano
Reducción cerrada bajo sedación y colocación de yeso o muñeca), subluxando con esta maniobra la articulación.
Reducción abierta y fijación con clavillos
Antibioticoterapia 18.¿Qué maniobra realizaría a la exploración para
Artrotomía de cadera, lavado y antibioticoterapia confirmar el cuadro?
Ninguna de las anteriores Tracción-contratracción
El tratamiento integral de la artritis séptica consiste en Colocación de vendaje elástico
lavado de la articulación y antibioticoterapia por un mes. Extensión y supinación
Pronación y valgo
CASO CLÍNICO 4
Ante la sospecha clínica de subluxación de la cabeza del
Femenino de 4 años traída por madre refiriendo posición radio, se debe hacer maniobra de supinación y extensión que
antiálgica de extremidad superior derecha. Niega no generan dolor. Habitualmente al realizarla reducen la
antecedente de traumatismo. A la exploración no se subluxación, ya sea realizada de manera activa o pasiva.
evidencía edema o deformidad de la extremidad, el dolor es
localizado a nivel de la cúpula radial. 19.¿Qué estudio de imagen es el indicado para el
diagnóstico?
16.Ante los datos clínicos del paciente, ¿cuál es su Rx AP y lateral de codo
principal sospecha diagnóstica? Rx AP y lateral de antebrazo
Fractura supracondílea humeral Tomografía axial de codo
Fractura de cúpula radial Ninguno
Codo de niñera
Fractura de Monteggia El diagnóstico del codo de niñera es clínico ya que no arroja
hallazgos patológicos en el estudio de imagen. Sólo se deben
Por la edad de la paciente y la presentación del cuadro, se realizar estudios de imagen cuando exista la duda sobre los
sospecha de subluxación de la cabeza del radio (Codo de diagnósticos diferenciales.
niñera), la cual produce dolor a la palpación sobre ella y
mínimo edema de estructuras circundantes. 20.¿Cuál sería el tratamiento recomendado para ésta
patología?
Analgésico y reposo por 2 días
Férula por 2 semanas 23.¿Por cuánto tiempo se debe llevar el tratamiento en
Aparato de yeso braquipalmar por 3 semanas este tipo de lesiones?
Reducción abierta y colocación de clavillos 2 semanas
3 semanas
Tras haber reducido la subluxación de la cabeza del radio, la 6 semanas
recomendación únicamente es prescripción de analgesia y 1 mes
reposo mínimo. Estas medidas son suficientes para limitar el
cuadro agudo, así como la educación a los padres para La recomendación para el tratamiento de fracturas
prevenir recidivas. unicorticales o incompletas es inmovilización completa, la
cual debe permanecer 3 semanas.
CASO CLÍNICO 5
24.¿Cuál es el principal factor que limita el
Masculino de 4 años que cae de plataforma de juegos de 1 desplazamiento de los fragmentos de fractura?
metro de altura con contusión directa en antebrazo derecho. El músculo supinador largo
A la EF con edema ++ y mínima deformidad de antebrazo del La membrana interósea
lado cubital. El vaso nutricio
El periostio
21.¿Qué estudio de imagen debe solicitar en el servicio
de urgencias? Debido a la anatomía pediatrica y fase de crecimiento óseo el
Rx AP y lateral de antebrazo derecho periostio es más grueso y el hueso más elástico, razón por la
Rx AP y lateral de muñeca derecha cual este tipo de fracturas tienen mejor pronóstico en niños.
Rx AP y lateral de húmero derecho
Rx AP y lateral de codo derecho 25.¿Cuál posible complicación se puede presentar de
ésta patología?
Para valorar una probable fractura, se deben solicitar 2 Detención del crecimiento
proyecciones en las que se visualice el segmento afectado Sobrecrecimiento
completo desde su articulación proximal hasta la distal. Deformidad angular
Todas las anteriores
22.En caso de encontrar una fractura incompleta, ¿cuál
es el tratamiento de elección? De acuerdo al grado de angulación o deformidad al
Uso de cabestrillo inmovilizar y por daño directo o indirecto al cartílago de
Uso de muñequera crecimiento pueden presentarse complicaciones como
Aparato de yeso braquipalmar detención del crecimiento, sobrecrecimiento, deformidad
Aparato de yeso antebraquipalmar angular, consolidación viciosa, entre otras, aunque el riesgo
de complicaciones es menor al 10%.
La realización de maniobra mínima de reducción e
inmovilización completa del segmento están asociadas con
excelente pronóstico en fracturas incompletas.
Pie plano / Epifisiolistesis femoral proximal / Enfermedad las siguientes corresponde a la explicación
de Osgood-Schlatter / Fracturas en edad pediátrica / fisiopatológica de la actitud de la extremidad
Lesiones tumorales afectada?
Apofisitis por tracción en disco de crecimiento
CASO CLÍNICO 1 Ruptura de las fibras de fibrocartílago
Desplazamiento posteroinferior de la epífisis sobre metáfisis
Un niño de 14 años acude a la consulta por dolor en cara Aumento de líquido sinovial articular
anterior de muslo y rodilla izquierda posterior a un partido
de fútbol jugado hace 15 días. No presenta ningún signo de La actitud de la cadera con flexión y rotación externa y
desarrollo puberal. Permanece con una actitud en flexión y limitación a la rotación interna, corresponde a un
rotación externa de la cadera y flexión de la rodilla, con deslizamiento de la capa proliferativa e hipertrófica del disco
limitación dolorosa a los intentos de movilización articular. de crecimiento.

1. ¿El primer diagnóstico de sospecha debe ser? 4. De acuerdo a los antecedentes y la exploración
Artritis séptica de cadera física ¿qué estudio radiográfico solicitaría para
Epifisiolistesis femoral proximal confirmar el diagnóstico y normar conducta de
Enfermedad de Perthes tratamiento?
Enfermedad de Osgood Schlatter Lowenstein
Falso perfil
En pacientes masculinos que durante la pubertad presentan Dunn
rotación de cadera limitada se debe sospechar deslizamiento Merchant
epifisiario femoral respecto al cuello femoral (epifisiolisteis
femoral proximal). Lo más adecuado ante sospecha de epifisiolistesis femoral
proximal es solicitar radiografías con una proyección de
2. ¿Cuál de las siguientes maniobras corresponde a su Lowenstein o rana, ya que esta posición es la mejor en
sospecha diagnóstica? imagen para medir y valorar el desplazamiento epifisiario
Puddu proximal femoral.
McMurray
Logg Roll 5. Tras la confirmación diagnóstica, ¿cuál sería su
Drennan conducta a seguir?
Lavado quirúrgico
La maniobra de Drennan corresponde a una limitación para Anclas todo dentro por vía artroscópica
realizar rotación interna y aducción de la cadera afectada Fijación in situ con tornillo
con tendencia a la flexión y rotación externa de la misma a la Observación solamente
exploración.
La fijación in situ ha demostrado ser la estrategia que logra
3. A la exploración física, se encuentran positivas las prevenir la progresión del deslizamiento epifisiario proximal.
maniobras que realiza según su sospecha. ¿Cuál de
CASO CLÍNICO 2 Línea interarticular medial rodilla
Cara anterior de muslo
Un niño obeso de 15 años se queja de dolor en la rodilla Polo Inferior de patela
izquierda desde hace 6 semanas, posterior a realizar Tuberosidad anterior de tibia
ejercicio de carrera. No presenta ocupación articular ni
bloqueos, ni refiere historia de traumatismos. Sólo refiere la El dolor a la digitopresión a nivel de la tuberosidad anterior
madre que en el último año, la talla del menor ha ido de la tibia es característico de la enfermedad de Osgood
aumentando de manera repentina, así como la disminución Schlatter.
del peso del menor.
9. ¿Cuál sería el tratamiento de elección indicado en
6. De acuerdo a la descripción del padecimiento, ¿cuál éste caso?
es la principal sospecha diagnóstica? Colocación de yesos seriados
Osteocondrosis de Sinding Larsen Johansson Banda restrictiva
Enfermedad de Perthes Disminuir actividad física
Enfermedad de Osgood Schlatter B y C son correctas
Bloqueo meniscal de la rodilla
En la enfermedad de Osgood-Schlatter, al ser una patología
La enfermedad de Osgood-Schlatter es un proceso que afecta que se autolimita, la banda y el reposo son medidas en
a los adolescentes en el periodo de crecimiento. Se trata de general que ayudan para mejorarlos síntomas. Normalmente
una enfermedad que consiste en el despegamiento del estas medidas bastan para el manejo.
cartílago de crecimiento de la tibia como consecuencia de la
realización de gestos repetidos. Se manifiesta por la aparición 10.A pesar del tratamiento que usted ofrece al menor,
de dolor en la cara anterior de la rodilla, por debajo de la la madre regresa a su consulta argumentando que
rótula y una masa palpable en la zona dolorosa. no hay mejoría de los síntomas. ¿Cuál es la
explicación que le diría a la madre con respecto al
7. ¿En qué relación de género se encuentra diagnóstico otorgado?
habitualmente ésta patología? Es una enfermedad que se autolimita al momento del cierre
Hombres 1:1 Mujeres de la fisis de crecimiento
Mujeres 2:1 Hombres No se toma de forma adecuada los medicamentos por ello
Hombres 3:1 Mujeres continua con el dolor
Hombres 2:1 Mujeres Necesita que baje de peso y acudir con un nutriólogo
Requiere de antibióticos para mejorar el edema de la rodilla
Este trastorno se observa principalmente en adolescentes
deportistas, generalmente entre los 10 y los 15 años. Es tres La enfermedad de Osgood-Schlatter suele ser autolimitada.
veces más común en varones. En pocos casos se requiere de tratamiento quirúrgico, ya que
con los cierres de los discos de crecimiento a los 14 años en
8. De acuerdo a la sospecha diagnóstica, ¿cuál es el niñas y a los 16 años en niños aproximadamente, se limita el
punto álgico de la exploración que confirma el cuadro.
diagnóstico?
CASO CLÍNICO 3
13.A las pocas horas es traído nuevamente a urgencias
Masculino de 6 años de edad cae de su bicicleta con debido a que se encuentra irritable refiriendo dolor
extremidad superior derecha en extensión presentando carga en el antebrazo. A la exploración, el llenado capilar
axial de la misma. Es llevado al servicio de urgencias con distal es normal. En la radiografía no se han
aumento de volumen y deformidad del antebrazo. producido cambios a nivel del foco de fractura, pero
presenta intenso dolor a la extensión pasiva de los
11.¿Con respecto al mecanismo de lesión, qué dedos y palidez de los dedos. ¿Cuál es el principal
porcentaje de los casos corresponde a lesión de signo que le alerte de una posible complicación?
región de la metáfisis distal? Irritabilidad del niño
62% Palidez de los dedos
20% Extensión pasiva de los dedos
14% Desviación de la fractura
10%
Debe explorarse la extensión de los dedos de la extremidad
Estadísticamente, el 62% de los casos de lesiones del afectada, ya que el dolor intenso a la extensión pasiva
antebrazo en niños mayores de cinco años corresponde a la muscular es normalmente el principal y primer signo que se
región de la metáfisis distal. presenta en un síndrome compartimental.

12.De manera inmediata solicita radiografías AP y 14.¿Cuál es, ante las siguientes, la actitud a seguir?
lateral del antebrazo afectado, observando fractura Mantener el miembro elevado y esperar
metafisiaria distal de radio y cúbito con angulación Abrir el yeso longitudinalmente
dorsal de 10 grados de cúbito y un cabalgamiento de Abrir la porción distal del yeso
1cm con angulación dorsal del radio, por lo que Retirar el yeso totalmente y elevar el miembro
decide realizarle manipulación de la misma con
tracción-contratracción bajo sedación colocándole Ante cualquier sospecha de síndrome compartimental se debe
un yeso braquipalmar, se toma control radiográfico retirar cualquier aparato o yeso que se encuentre en el sitio
observando adecuada reducción de las fracturas. de edema.
¿Cuál es la complicación más temida de dicho
tratamiento? 15.De los siguientes signos, ¿cuál no corresponde a
Síndrome compartimental complicación por síndrome compartimental?
Extensión de la fractura al momento de la manipulación Isquemia cutánea
Dedos equimóticos por la manipulación Hipoestesia en el trayecto del nervio afectado
No reducción del cabalgamiento óseo con la manipulación Ausencia de pulso
Cúbito valgo
El aumento de la presión intracompartimental puede
presentarse con la manipulación y se agrava con la colocación A la exploración de un paciente con síndrome compartimental
de un yeso justo y mantener la extremidad hacia abajo, en antebrazo, se podrá encontrar isquemia cutánea,
representando una complicación grave.
hipoestesia, dolor al estiramiento pasivo muscular y pulsos
disminuidos o ausentes. El arco longitudinal está formado entre el extremo posterior
del calcáneo y las cabezas de los metatarsianos. Es más alto
en su cara medial, donde forma la parte medial del arco
CASO CLÍNICO 4 longitudinal, y más bajo en su cara lateral, donde constituye
la parte lateral. El músculo tibial posterior en su inserción en
Femenino de 2 años 3 meses de edad es llevada a consulta de el escafoides contribuye a su mantenimiento.
ortopedia por madre referida desde su pediatra por
presentar pie plano desde hace 6 meses que inició la marcha
independiente, refiere que previamente no presentaba dicha 19.¿Qué estudio de imagen solicitará posteriormente
deformidad. para valorar la patología?
RX DP y oblicua de ambos pies en reposo
16.Ante los datos clínicos de la paciente ¿Qué variedad RX DP y oblicua de ambos pies en decúbito
de la enfermedad se está presentando? RX DP y lateral de ambos pies en reposo
Rígido RX DP y lateral de ambos pies en carga
Mecánico
Teratológico El estudio de imagen recomendado es son las radiografías en
Flexible dos proyecciones (AP y lateral) con carga, de ambos pies.

El pie plano valgo flexible es la variedad más común y 20.¿Cuál sería el tratamiento inicial recomendado para
aparece al realizar carga sobre la extremidad, como es el ésta patología?
caso en esta paciente que se presentó al iniciar la marcha. Férula por las noches
Plantillas de cuero 3 horas diarias
17.¿Qué observación debe realizar a la madre acorde a Ejercicios de fortalecimiento
la edad del paciente? Calzado ortopédico permanente
Debe usar calzado ortopédico
Debe iniciar uso de plantillas Para esta paciente, la única recomendación por el momento
Aun no es valorable hasta los 3 años consiste en el fortalecimiento del músculo tibial posterior con
Debe valorarse cada mes ejercicios, y revaloración posterior.

Se debe comentar a la madre que la deformidad no es CASO CLÍNICO 5


enteramente valorable antes de los 3 años de edad.
Masculino de 15 años que acude al centro de salud refiriendo
18.¿Qué músculo es el principal encargado de dolor en tibia proximal de 1 mes de evolución secundario a
mantener el arco longitudinal medial? traumatismo directo en cara anterior de rodilla. EF con
Peroneo largo mínimo aumento de volumen respecto a contralateral, con
Flexor común de los dedos dolor a palpación de metáfisis tibial, mínimo derrame
Tibial anterior articular y arcos de movimiento completos.
Tibial posterior
21.¿Qué estudio de imagen debe solicitar? A y B son correctas
Rx AP y lateral de rodilla
Rx AP Y lateral de Fémur La reacción perióstica y aumento de volumen de partes
Ultrasonido de rodilla blandas, junto con los bordes mal definidos, son hallazgos
Resonancia magnética de rodilla radiográficos comúnmente encontrados en las lesiones
malignas.
Con el objetivo de valorar y descartar lesión ósea, se deben
solicitar radiografías en dos proyecciones (AP y lateral) del 25.En caso de tratarse de una lesión maligna ¿Cuál
segmento afectado. posible complicación se puede presentar en ésta
patología?
22.En caso de encontrar una lesión ósea, ¿cuál debe ser Diseminación local y a distancia
su conducta? Sobre crecimiento
Colocar Férula de reposo Deformidad angular
Indicar analgésico Detención del crecimiento
Referencia a especialista
Cita en 1 mes para vigilancia y evolución Dentro de las complicaciones probables que puede tener un
tumor óseo se encuentra la diseminación local y diseminación
El caso especifica que el paciente llega a un primer nivel de a distancia si no es detectado a tiempo y recibe un
atención, en caso de encontrar lesión ósea, no es necesario tratamiento oportuno.
ningún tratamiento de urgencia en el momento, ya que es
una lesión con un mes de evolución. Lo indicado es referir a CASO CLÍNICO 5
segundo nivel de atención.
Paciente masculino de 10 años que es llevado a consulta por
23.Ante una imagen radiológica de bordes mal la madre refiriendo tener el pie plano. Al interrogatorio
definidos ¿cuál debe ser su sospecha diagnóstica? dirigido se encuentra asintomático.
Defecto de nacimiento
Secuelas de fractura no diagnosticada 26.¿Qué observación sobre la patología debe explicarle
Lesión maligna a la madre?
Infección no diagnosticada Es patología que debe ser valorada sólo por especialista en
ortopedia
Una de las características que deben hacernos pensar que se No es valorable hasta la madurez esquelética
trata de una lesión maligna, es la mala definición de los Debe usar calzado ortopédico
bordes en la imagen. No requiere tratamiento

24.¿Cuál de las siguientes características radiológicas Normalmente cuando el pie plano se encuentra en su
indicarían malignidad? variedad asintomática no requiere tratamiento alguno.
Reacción perióstica
Compromiso de partes blandas 27.¿Cuál es la presentación clínica más común en el pie
Presencia de callo blando plano?
Dolor tras realizar deporte En los pacientes con pie plano, la exploración física completa
Cansancio tras caminar que incluya análisis de la marcha y visualización con
Dolor tras bipedestación prolongada plantoscopio son las principales herramientas diagnósticas y
Ninguna de las anteriores de seguimiento.

La presentación clínica más común en el pie plano es


primordialmente asintomática. Síndrome de túnel del carpo / Tenosinovitis de Quervain /
Lesiones deportivas / Fracturas
28.¿Cuál es el principal factor de riesgo para el
desarrollo síntomas en el pie plano? CASO CLÍNICO 1
Genu valgo
Obesidad Femenino de 55 años, fuma 1 cajetilla de 20 cigarrillos
Genu varo diario. Acude a consulta refiriendo dolor en primer dedo de
Valgo de retropié mano derecha de 2 meses de evolución irradiado de la cara
dorsal del pulgar hasta la muñeca, se incrementa tras la
La obesidad tiene implicaciones mecánicas directas e actividad física manual (carpintería). Niega pérdida de peso
indirectas para la presencia de dolor y fatiga en pacientes o fiebre. A la EF leve edema y dolor a palpación del dorso del
con pie plano y otras condiciones ortopédicas. primer dedo hasta tabaquera anatómica.

29.¿Cuál no es un componente del tratamiento 1. De acuerdo a su sospecha diagnóstica, ¿qué


conservador del pie plano? antecedente referido es de importancia?
Ejercicios de estiramiento Antecedente laboral
Uso de calzado con tacón Sedentarismo
Ejercicios de fortalecimiento Tabaquismo
Uso de ortesis (plantillas) Pérdida de peso

El acortamiento o contractura de los gemelos y el sóleo son En las enfermedades inflamatorias degenerativas los
los componentes del pie plano por lo que el calzado con antecedentes laborales son el principal factor predisponente.
tacón no está indicado en tratamiento ya que con su uso, se
favorecen estas condiciones. 2. ¿Cuál es su sospecha diagnóstica?
Neuroma de nervio radial
30.¿Qué medida se debe realizar de rutina en el Tenosinovitis de Quervain
paciente con pie plano? Síndrome de túnel del carpo
Visualización de huella en plantoscopio Dedo en resorte
Rx Dorsoplantar y lateral de pie
Análisis de marcha La tenosinovitis de Quervain es una tendinitis que afecta a los
A y C son correctas tendones que ocupan el primer compartimento extensor de la
mano, abductor largo y extensor corto del pulgar,
produciendo dolor con los movimientos del pulgar. El dolor
por actividad física intensa o repetitiva del primer dedo en el CASO CLÍNICO 2
dorso es característico de esta entidad.
Femenino de 53 años de edad, secretaria de escuela
3. ¿Qué músculos conforman el origen del dolor? secundaria que acude a consulta refiriendo desde hace 2
Abductor largo y extensor corto del pulgar meses debilidad en mano derecha acompañada de
Extensor común de los dedos adormecimiento ocasional. Refiere dejar caer objetos por
Flexor largo y oponente del pulgar falta de fuerza en 2 ocasiones. Niega antecedentes
Aductor largo y oponente del pulgar traumáticos, esguinces o fracturas previas. Cuenta con RX AP
y lateral de muñeca y mano de hace un mes en las cuales no
La tenosinovitis de Quervain es una tendinitis que afecta a los se evidencia lesión ósea. EF con peso de 87 kg y talla de
tendones que ocupan el primer compartimento extensor de la 1.57m, signos vitales normales.
mano, abductor largo y extensor corto del pulgar,
produciendo dolor con los movimientos del pulgar. 6. De acuerdo a la descripción del padecimiento ¿Cuál
es la principal sospecha diagnóstica?
4. ¿Cuál maniobra clínica confirmaría el diagnóstico? Compresión nerviosa periférica
Fallen Enfermedad reumatológica
Allen Cuadro infeccioso
Filkenstein Fractura por estrés
Duncan
Por la ocupación y el cuadro de la paciente, se sospecha de
La maniobra de Finkelstein consiste en provocar el dolor de síndrome de túnel del carpo, que es la principal causa de
forma selectiva al ejecutar un movimiento de desviación dolor en muñeca. Se presenta con datos de compresión
cubital de la muñeca con el pulgar en flexión completa y el nerviosa por atrapamiento del nervio mediano.
resto de los dedos libres. Es positiva si provoca o aumenta de
forma significativa el dolor que sufre el paciente La maniobra 7. ¿Qué dedos esperaría encontrar con hipoestesia en
de Filkenstein es patognomónica de Quervain. caso de compresión de nervio mediano?
Dorso de 1er y 2do dedos
5. ¿Cuál es la etiología de este padecimiento? 4to y mitad cubital de 5to dedos
Idiopática Cara anterior de 1ro, segundo y 3er dedo
Iatrogénica Dorso de 2do, 3ero y mitad cubital de 4to dedo
Inflamatoria
Neoplásica El nervio mediano es un nervio mixto. Dentro de sus funciones
sensoriales en la mano, da lugar a la rama cutánea palmar,
La tenosinovitis de Quervain es una entidad de origen que inerva la parte lateral de la palma, y la rama cutánea
inflamatorio, a consecuencia del trauma repetitivo y digital, que inerva los tres dedos laterales y medio en la
sobreuso. superficie anterior de la mano.
8. ¿Qué dedos esperaría encontrar con hipoestesia en CASO CLÍNICO 3
caso de compresión de nervio cubital?
Dorso de 1er y 2do dedos Masculino de 21 años de edad, sin antecedentes personales
4to y mitad cubital del 5to dedo de importancia. Acude al servicio de urgencias posterior a
4to y mitad cubital del 5to dedo sufrir caída de bicicleta durante carrera en el centro de la
Cara anterior de 1ro, segundo y 3er dedo ciudad, recibe contusión directa en hombro izquierdo.
Dorso de 2do, 3ero y mitad cubital de 4to dedo Refiere dolor intenso en hombro izquierdo que incrementa
con la movilización. A la EF con edema y dolor en
El nervio cubital cubre tanto funciones motoras como articulación acromioclavicular, no equimosis, los arcos de
sensoriales. Inerva y da función motora a diferentes músculos movilidad son completos con dolor intenso en los últimos
del antebrazo y mano. En cuanto a la función sensitiva, grados, con signos vitales en parámetros normales.
proporciona inervación y percepción en los costados palmar y
dorsal del quinto y cuarto dedos. 11.¿Qué diagnóstico es el más probable ante este
cuadro?
9. ¿Qué elemento no forma parte del túnel carpiano? Fractura de clavícula
Nervio cubital Luxación glenohumeral
Nervio mediano Luxación acromioclavicular
Flexor común superficial de los dedos Luxación esternoclavicular
Flexor común profundo de los dedos
La luxación acromioclavicular es una de las lesiones
El túnel del carpo incluye al ligamento transverso del carpo, traumáticas más frecuentes en los jóvenes deportistas, con
el nervio mediano, los tendones flexores, las vainas de los presencia daño de los medios de unión de la articulación y
tendones y los huesos del carpo. El nervio cubital no es parte separación de las superficies articulares. El mecanismo de
de esta estructura. trauma y el dolor localizado generan la sospecha diagnóstica.

10.¿Cómo está conformada la hilera proximal de los 12.¿Qué signo clínico encontramos positivo en estos
huesos del carpo? pacientes?
Trapecio, Trapezoide, Grande y Ganchoso Charretera
Trapecio, Trapezoide, Piramidal y Pisiforme Sulcus
Escafoides, Piramidal, Grande y Ganchoso Jobe
Escafoides, Semilunar, Piramidal y Pisiforme Tecla

Los huesos del carpo son: fila proximal- escafoides, El signo de la tecla es positivo en presencia de luxación
semilunar, piramidal y pisiforme; fila distal- trapecio, acromio clavicular debido a la inestabilidad de la articulación
trapezoide, hueso grande y hueso ganchoso. acromio clavicular. Con la palpación se puede deprimir y
recolocar la clavícula en la articulación, dando lugar a este
signo.

13.¿Qué estudio solicita para confirmar diagnóstico?


Rx AP de hombro deformidad de hombro con integridad neurológica distal e
Rx tangencial de escápula hiperextensión de codos y primer dedo. Signos vitales: TA
TAC de hombro 110/70mmHg, FC 68, Temp. 36.5 °C, 1.72m, 82kg.
RM de hombro
16.Ante los datos clínicos del paciente, ¿cuál es su
Normalmente basta con una Rx AP de hombro para confirmar principal sospecha diagnóstica?
el diagnóstico de luxación acromioclavicular, ya que en este Luxación acromioclavicular
estudio se muestra la alineación de la articulación Fractura de clavícula
acromioclavicular. Luxación glenohumeral
Fractura de diáfisis humeral
14.¿Qué tratamiento es el recomendado con un
desplazamiento mínimo? La luxación glenohumeral es una lesión frecuente en la
Analgésico e inmovilización completa práctica deportiva y se produce cuando la cabeza del húmero
Analgésico y reposo se luxa del glenoides, ocasionando un intenso dolor que
Analgésico y movilización temprana impide mover el brazo. Causa deformidad y dolor intenso del
Quirúrgico hombro, el mecanismo de lesión más frecuente es en
abducción y rotación externa.
Ante una luxación acromioclavicular con desplazamiento
mínimo, solamente se indica reposo con uso de cabestrillo y 17.¿Cuál es el factor de riesgo presente en esta
prescripción de analgésicos para el control del dolor. paciente para el desarrollo de esta enfermedad?
Obesidad
15.¿En qué se basa la clasificación de Rockwood? Hiperlaxitud
Forma residual del acromion Actividad física
Desplazamiento de la clavícula Edad
Situación de la cabeza humeral
Desplazamiento de la glenoides Los datos que se encontraron en la exploración física, son
congruentes con hiperlaxitud ligamentaria (hiperextensión de
La clasificación de Rockwood se basa en la situación y pulgar y codos). Este puede ser un factor de riesgo para
desplazamiento de la clavícula con respecto al acromion. De presentar luxaciones como la que presenta en esta ocasión.
esta manera, se identifican seis tipos de luxaciones
acromoclaviculares. 18.¿Qué nombre recibe la deformidad característica
que produce esta lesión?
Tecla
CASO CLÍNICO 4 Charretera
Sulcus
Femenino de 17 años de edad es llevada al servicio de Hill-Sachs
urgencias posterior al sufrir caída hacia atrás al jugar
básquetbol, presentando deformidad, dolor intenso e El signo de la charretera es típico de la luxación
incapacidad funcional del miembro torácico derecho. A la EF glenohumeral anterior. Desaparece la redondez del hombro
por desplazamiento de la cabeza humeral y se hace observamos que ésta se proyecta a la altura de la estiloides
prominente la punta del acromion. cubital. Se percibe el pulso radial y la sensibilidad es normal.

19.¿Cuál es el tratamiento inmediato tras el diagnóstico 21.¿Qué estudio de imagen debe solicitar para
clínico-radiológico? complementar diagnóstico?
Reducción abierta y osteosíntesis Rx DP y lateral de muñeca
Reducción cerrada e inmovilización Rx DP y lateral de antebrazo
Reducción abierta e inmovilización Rx AP y lateral de húmero
Movilidad activa y pasiva continúa Rx lateral y oblicuas de muñecas

La reducción cerrada bajo sedación en el menor tiempo Se deben solicitar 2 proyecciones (dorsopalmar y lateral de
posible es lo indicado en la luxación de la articulación muñeca) para confirmar diagnóstico y evaluar
glenohumeral para disminuir el riesgo de necrosis por desplazamientos.
isquemia de la cabeza humeral.
22.La radiografía muestra una fractura de la
20.¿Qué presentación es la más común de esta extremidad distal del radio, a 1,5 cm. de la
patología? interlínea radio-carpiana. El macizo epifisiario está
Anterior desplazado hacia afuera con una penetración de la
Posterior diáfisis en la epífisis. Existe una diástasis de 5 mm
Inferior entre la cara interna del radio y la cabeza cubital.
Medial En la radiografía de perfil la epífisis está desplazada
en dirección dorsal y proximal. ¿Cuál es la principal
Según los datos de la GPC mexicana, más del 95% de las medida que se debe tomar para evitar el edema?
luxaciones glenohumerales son anteriores. La rotación Reducción abierta inmediata
externa forzada en abducción desplaza la cabeza del húmero Colocación de férula posterior
fuera de la cavidad glenoidea, avulsionando hueso y tejidos Mantener la extremidad elevada
blandos de las estructuras del proceso anterior. Calor local

Con fines de evitar el edema en la extremidad afectada, se


CASO CLÍNICO 5 debe mantener la extremidad elevada, ya que esta maniobra
evita hasta cierto punto la congestión por gravedad.
Mujer de 30 años, sufrió una caída mientras patinaba (1 hora
de evolución) y actualmente presenta dolores en la muñeca. 23.La radiografía muestra una fractura de la
Al examen físico, usted nota una curvatura dorsal y una extremidad distal del radio, a 1,5 cm. de la
inclinación radial de la mano. Los movimientos de flexión interlínea radio-carpiana. El macizo epifisiario está
están un poco limitados por el dolor, sobre todo la desplazado hacia afuera con una penetración de la
pronosupinación. La palpación muestra un punto doloroso a 2 diáfisis en la epífisis. Existe una diástasis de 5 mm
cm. por encima de la estiloides radial (proximal) y entre la cara interna del radio y la cabeza cubital.
En la radiografía de perfil la epífisis está desplazada
en dirección dorsal y proximal. ¿Cuál es el CASO CLÍNICO 6
tratamiento de elección de acuerdo a los hallazgos
radiográficos? Masculino de 25 años de edad que es llevado al servicio de
Reducción abierta y fijación interna urgencias por presentar dolor intenso y derrame articular
Reducción cerrada y colocación de aparato de yeso posterior a realizar flexión forzada tras realizar barrida al
Artrodesis jugar fútbol, el dolor se intensifica conforme avanza el
Analgésico y férula posterior partido hasta tornarse intenso y le impide el apoyo.

Debido a que el trazo de la fractura es extrarticular el 26.¿A qué se debe el derrame articular en este
tratamiento de elección consiste en realizar una reducción padecimiento?
cerrada e inmovilización de la extremidad con yeso. Sinovitis
Hemartrosis
24.Además de la elevación de la extremidad, ¿qué otra Reacción Inflamatoria
medida deberá realizar el paciente en casa? Mecánica
Calor local sobre inmovilización
Reposo absoluto de la extremidad Por el mecanismo de la lesión y la presentación clínica, la
Movilización continúa de los dedos sospecha es de lesión de cuerno posterior de menisco por
Retiro intermitente de la inmovilización flexión forzada El derrame articular se debe a una reacción
inflamatoria por esta afección.
No se recomienda el retiro de la inmovilización, la aplicación
de calor ni el reposo total de la extremidad. Lo indicado es 27.¿Cuál es el diagnóstico más probable con esta
sugerir la movilización constante de los dedos, ya que la clínica?
contracción muscular isotónica evita la estasis venosa y Lesión de ligamento colateral medial
disminuye el riesgo de edema. Lesión de ligamento colateral lateral
Lesión de retináculo lateral
25.¿Cuál es la complicación que se puede presentar en Lesión meniscal
estos casos?
Consolidación viciosa El posible mecanismo de lesión, y el empeoramiento
Rigidez articular progresivo del cuadro, nos hacen sospechar de lesión de
Síndrome de túnel carpiano cuerno posterior de menisco por flexión forzada.
Todas las anteriores
28.¿Qué maniobra reafirmará la sospecha diagnóstica?
La distrofia simpática refleja, consolidación viciosa, rigidez Mc Murray
a r t i c u l a r, s í n d r o m e d e t ú n e l c a r p i a n o , s í n d r o m e Cajón
compartimental, entre otros, son posibles complicaciones de Pivot shift
una fractura de muñeca. Ficat

Ante la sospecha de lesión meniscal se deben aplicar las


maniobras de Mc Murray, Appley y Steinman, que son las
maniobras diagnósticas para lesión meniscal. La maniobra de ni lavado articular, ni iniciar terapia física o ejercicio de
Mc Murray, se explora colocando al paciente en decúbito inmediato.
supino, se flexiona la rodilla y se coloca el dedo en el borde
del menisco, seguido de una extensión en rotación interna y
luego en rotación externa. Si existe una lesión del menisco EXAMEN FINAL
interno se notará un chasquido al extender en rotación
externa, y si existe una lesión del menisco externo el CLINICAL CASE 1
chasquido se oirá al extender en rotación interna.
A 15-year-old soccer player runs into a goal post, protecting
29.¿Qué signo clínico puede encontrar positivo acorde his body with an outstretched left hand. After the game he
al mecanismo de lesión? complains to his father that his wrist hurts and they stop by
Lachman the local emergency department. The boy screams in pain
Bostezo medial when the emergency physician palpates the floor of the
Bostezo lateral anatomic snuffbox.
Appley
1. What is the most likely diagnosis?
Ante la sospecha de lesión meniscal se deben aplicar las First metacarpal fracture
maniobras específicas para menisco. Mc Murray, Appley y Distal Ulnar fracture
Steinman son las maniobras diagnósticas para lesión meniscal. Radial styloid fracture
La maniobra de Appley se realiza con el paciente en decúbito Scaphoid fracture
prono y se compara el dolor que provoca la flexo-extensión
con la rodilla a compresión con la rodilla a distracción, todo The most commonly fractured bone of the hand is the carpal
ello imprimiendo un movimiento rotatorio a la pierna, scaphoid, and it causes pain at the bottom of the anatomic
internamente para explorar el menisco externo y snuffbox, such as pain referred by this patient.
externamente para explorar el interno (el talón del paciente
señala el menisco explorado). 2. What is the election study at emergency room?
X-ray films of left wrist
30.¿En qué consiste en tratamiento inmediato de ésta Wrist ultrasound
lesión? Hand and wrist CT
Artrocentesis, inmovilización y lavado articular Wrist MRI
Analgésico, inmovilización y medios físicos
Analgésico, terapia física y ejercicio de impacto The election study that should be requested at the
A y C son correctas emergency room is a simple x-ray film in two projections of
the affected wrist.
El tratamiento inicial en caso de lesión del menisco consiste
en control de dolor, estabilización de la articulación y 3. If the study of the left wrist and hand shows no
disminución de edema por medios físicos. Se descartan las fracture. What is the best next step in management?
otras opciones, ya que no está indicado realizar artocentesis Removable plaster splint and physical therapy
Rest, ice, and no steroidal anti-inflammatory medications
Short arm cast and physical therapy in 4–6 weeks it has started to shoot past his knees. He states that the pain
Thumb spica cast is worst with walking, and he denies any recent trauma or
illness. On physical examination his lower extremities are
Even though x-ray films don´t show a fracture, this patient neurovascularly intact, with full 5/5 strength, intact
should be treated with immobilization of the affected sensation, and 2+ dorsalis pedis and posterior tibial pulses.
extremity with a thumb spica cast. No extra studies are
necessary at this moment, but x-ray films should be repeated 6. Which is the most appropriate next step in
after two weeks. diagnosis?
CT of the spine
4. The study of the left wrist and hand shows no Electromyography of the lower extremities
fracture. What would you do to confirm diagnosis? Lumbar spine plain films
Hand and wrist MRI MRI of the lower back
Repeat x-ray films of the left wrist and hand in 2–3 weeks
Hand and wrist ultrasound A MRI of the lower back should be performed to this patient.
Repeat physical examination after 5 days In presence of radiculopathy, MRI is the gold standard to
diagnosis.
A simple wrist x-ray doesn’t always show fracture, but in
presence of clinical suspect, you must immobilize and repeat 7. In addition to analgesia, which of the following is
x-ray films after two weeks. the most appropriate next step in management?

5. What is the name of the most common complication Full bed rest
of this pathology? X-rays of the spine
Non-union (pseudarthrosis) Regular activity
Wrist instability Lumbar spine traction
Radio-ulnar instability
Semilunar luxation Regular light activity should be recommended to this patient,
because this helps to improve further mobility and speeds up
Due to its poor vascularization, pseudarthrosis is the most recovering.
common complication of a scaphoid fracture.
8. In case of radiculopathy, which is the most affected
intervertebral discoid space?
CLINICAL CASE 2
L2 – L3
A 75-year-old white man presents to his primary care L3 – L4
physician complaining of lower back and leg pain. The pain is L4 – L5
brought on by walking, has been steadily worsening over the L5 – S1
past few years, and is not relieved by no steroidal anti-
inflammatory drugs. In the past the pain was confined only
to his lower back and upper leg/thigh, but in recent months
Statistics show that L4-L5 lumbar space is the most affected 11.En caso de no presentar mejoría clínica tras el
spine level when radiculopathy is present, followed by L5-S1. tratamiento inicial ¿cuál debe ser la conducta a
seguir?
Realizar punción con anestésico local en sitio de fractura
CASO CLÍNICO 3 Realizar fasciotomía
Realizar manipulación de trazo de fractura
Un niño de 8 años presenta una fractura mínimamente Realizar exploración vascular
desplazada de cúbito y radio izquierdos que es tratada
mediante inmovilización con yeso cerrado, manteniendo el En un síndrome compartimental, al no presentar mejoría con
codo en 90º de flexión. A las pocas horas es traído a tras la apertura del yeso y medidas generales, se debe
urgencias debido a que se encuentra irritable, refiriendo proceder a realizar fasciotomías del segmento afectado.
dolor en el antebrazo. A la exploración, el relleno capilar es
normal. En la radiología no se han producido cambios a nivel
del foco de fractura, pero presenta intenso dolor a la CASO CLÍNICO 4
extensión pasiva de los dedos.
Mujer de 60 años sin antecedentes patológicos previos, sufre
9. ¿Cuál es la sospecha diagnóstica? caída en la ducha de su domicilio, presentando fractura
Síndrome compartimental subcapital desplazada de cuello femoral izquierdo, refiere
Lesión nerviosa hace una semana haber iniciado con terapia de reemplazo
Lesión vascular hormonal e ingesta de calcio y alendronato de sodio. A la EF
Lesión perióstica postrada en cama con equimosis en cadera izquierda y
miembro pélvico en rotación externa sin acortamiento.
El dolor intenso que no mejora con analgésicos y que
incrementa a la extensión pasiva de los dedos son los 12.¿Cuál es el diagnóstico más probable?
primeros datos de instauración de síndrome compartimental. Secuelas de displasia de cadera
Necrosis avascular de cadera
10.¿Cuál es la actitud a seguir? Fractura de cadera
Mantener el miembro elevado y esperar Luxación traumática de cadera
Abrir el yeso longitudinalmente y elevar el miembro
Abrir la porción distal del yeso La presencia de dolor intenso y equimosis son sugerentes de
Aumentar la dosis de analgésicos fractura de cadera. La ausencia de acortamiento
probablemente se debe a la localización intracapsular de la
La primera medida a realizar ante un probable síndrome fractura.
compartimental es disminuir la presión de los
compartimentos mediante la apertura longitudinal completa 13.¿Cuál de los siguientes es el principal factor de
del aparato de inmovilización y acolchonamiento. riesgo en esta paciente?
Edad
Ingesta de calcio
Terapia de reemplazo hormonal
Ingesta de antirresortivo óseo Meniscal del menisco lateral
Del ligamento cruzado posterior
Las mujeres postmenopáusicas incrementan en riesgo de Del ligamento colateral medial
fracturas por traumas de baja energía conforme aumenta la
edad. Los mecanismos torsionales son la principal causa de lesión
meniscal y el más lesionado comúnmente por sus
14.¿Cuál es la finalidad del tratamiento quirúrgico? características anatómicas es el menisco medial.
Movilización temprana
Prevenir úlceras por decúbito CASO CLÍNICO 6
Prevenir trombosis Paciente varón de 22 años de edad con hemartrosis en la
Todas las anteriores rodilla tras un traumatismo directo por la rodilla de un
compañero jugando al fútbol. Le causa dolor intenso y le
La prevención de complicaciones y mejorar el pronóstico para impide terminar de jugar el partido.
la vida y la función son los objetivos del tratamiento 17.¿Cuál es la lesión más frecuente que hay que
quirúrgico en fractura de cadera. sospechar?
Ruptura de ligamento cruzado anterior
15.¿Cuál de los siguientes tratamientos es el más Ruptura de la arteria poplítea
adecuado? Fractura del cuello del peroné
Prótesis total de cadera Lesión de la placa sinovial
Yeso pelvi-podálico
Osteosíntesis con tornillos La principal causa de hemartrosis traumática en jóvenes
Carga inmediata atletas es la ruptura del ligamento cruzado anterior. Dado el
mecanismo de lesión, y la presentación actual de este
Esta paciente requiere ser intervenida quirúrgicamente, con paciente, esta es la principal sospecha diagnóstica.
colocación de prótesis, debido al daño de la vascularidad que
sufre la cabeza del fémur en las fracturas de cuello. CASO CLÍNICO 7

Paciente masculino de 20 años de edad, es traído al servicio


CASO CLÍNICO 5 de urgencias por paramédicos por haber sufrido caída de su
motocicleta. Se obtiene una radiografía de brazo izquierdo
Un paciente de 20 años, jugando al fútbol, sufre una torsión en la que se observa fractura diafisaria del húmero.
en su rodilla al clavar los tacos en el césped y girar
bruscamente el tronco. La rodilla está dolorosa en el tercio 18.¿Qué nervio se ve ocasionalmente lesionado en
posterior de la interlínea articular medial y al forzar la asociación a fracturas diafisarias del húmero?
flexión, no presenta inestabilidades ligamentaria y se
aprecia un derrame articular escaso. Circunflejo
Radial
16.El diagnóstico más probable es lesión: Mediano
Meniscal del menisco medial
Cubital paciente es menor a 3 meses y que no puede realizar marcha,
se considera aguda inestable
El nervio radial discurre por el canal de torsión de la diáfisis
humeral, por lo que frecuentemente se lesiona en las 21.El tratamiento profiláctico en este paciente
fracturas. correspondería a:
Aplicación intraartricular de Itrio
Colocación de aparato de yeso
CASO CLÍNICO 8 Antibioticoterapia intravenosa
Fijación de cadera contralateral
Un muchacho obeso de 14 años acude a la consulta por dolor
en cara anterior de muslo y rodilla izquierdos desde un En presencia de factores de riesgo para epifisiolisis femoral
partido de fútbol jugado 15 días antes. No presenta ningún proximal, se debe fijar la cadera contralateral aún en
signo de desarrollo puberal. Permanece en silla de ruedas ausencia de síntomas o cambios radiológicos.
con una actitud en flexión y rotación externa de la cadera y
flexión de la rodilla, con limitación dolorosa a los intentos
de movilización articular. CASO CLÍNICO 9

19.El primer diagnóstico de sospecha debe ser: Paciente femenino de 18 meses, hijo de una madre
Epifisiolisis femoral proximal primigesta de embarazo post-término, sin adecuado control
Artritis séptica de cadera prenatal, con peso al nacer de 3200 gr y Apgar 8/9, es traída
Artritis reumatoide juvenil a consulta por cojera desde que empezó a caminar a los 16
Enfermedad de Legg-Calvé-Perthes meses. A la exploración presenta marcha en Trendelemburg,
con temperatura de 36.7 °C, resto de signos vitales
En pacientes masculinos obesos durante la pubertad, que normales.
presentan rotación de cadera limitada se debe sospechar
deslizamiento epifisiario femoral respecto al cuello femoral 22.Hay que sospechar en primer lugar en:
(epifisiolisis femoral proximal) como primera posibilidad Luxación congénita de cadera
diagnóstica. Sinovitis transitoria de cadera
Artritis séptica de cadera
20.C o r r e s p o n d i e n d o a l a c l a s i f i c a c i ó n d e l a Parálisis cerebral infantil
enfermedad:
No desplazada En presencia de claudicación por acortamiento de miembro
Inespecífica pélvico en infantes que inician la marcha debe descartarse
Aguda inestable luxación congénita de cadera, especialmente en pacientes
Velonodular del sexo femenino y con algún factor de riesgo para la
presencia de esta entidad.
Por la duración de los síntomas, la epifisiolisis femoral
proximal se clasifica como predeslizamiento, aguda, aguda 23.El estudio que debe solicitar para confirmar el
sobre crónica o crónica. Debido a que la duración en este diagnóstico será:
Artrocentesis Punción diagnóstica y terapéutica
Ultrasonido de cadera Ejercicios de fortalecimiento de tobillo
Rx de pelvis Inmovilización con aparato de yeso
TC de cadera Estabilización quirúrgica

A partir de los 3 meses de vida el estudio de imagen de En caso de fracturas con desplazamientos menores a 2 mm y
elección para el diagnóstico de luxación congénita de cadera sobre todo en lesiones extra articulares el tratamiento de
es la radiografía de pelvis. En pacientes menores a tres elección es conservador con inmovilización del segmento de
meses, el estudio de elección es el ultrasonido de cadera. la fractura.

26.¿Por cuánto tiempo debe aplicarse el tratamiento de


CASO CLÍNICO 10 elección?

Masculino de 11 años de edad que hace 2 días recibe 10 días


contusión directa con pie de otro compañero al jugar soccer, 2 semanas
le condiciona dolor intenso e incapacidad para el apoyo. A la 3 a 4 semanas
exploración con mínimo edema en tercio distal de pierna 8 a 12 semanas
derecha, con movilidad de tobillo completa, no
inestabilidad. Este paciente requiere inmovilización de la extremidad. La
inmovilización en fracturas en niños es de 3 semanas.
24.El estudio complementario de este paciente deberá
incluir: 27.El pronóstico funcional a largo plazo de este
paciente es:
Tomografía simple y contrastada de tobillo
Tomografía simple de pierna Bueno para la vida y la función
Radiografía simple de pierna Bueno para la vida y limitado para la función
Ultrasonido de tobillo y pierna Bueno para la vida y malo para la función
Malo para la vida y la función
La radiografía simple es el estándar para la detección de
lesiones óseas traumáticas, la tomografía tiene indicaciones El pronóstico de fracturas de este tipo en niños es bueno ya
e s p e c i f i c a s e n c a s o d e c o m p r o m i s o a r t i c u l a r, que no deja secuela funcional y habitualmente no amerita
multifragmentación o compromiso vascular. rehabilitación especifica.

25.La radiografía de este paciente muestra una pérdida CASO CLÍNICO 11


de continuidad de la cortical de la tibia sin
desplazamiento, el tratamiento de elección en este Femenino de 48 años, contador público, con 20 días de dolor
caso es: en base del primer dedo de mano derecha que incrementa de
manera progresiva hasta tornarse intenso y limita la función
del dedo, advirtiendo crepitación con cada movimiento. A la CASO CLÍNICO 12
exploración usted percibe crepitación con la flexo-extensión
en la superficie palmar de la base del dedo. Masculino de 6 años que sufre caída de brincolín con
contusión directa en mano, con muñeca y codo en extensión,
28.El diagnóstico clínico de esta paciente será: presentando dolor intenso y aumento de volumen en codo.
Túnel del carpo
Tenosinovitis de Quervain 31.La lesión a sospechar por frecuencia de aparición y
Dedo en gatillo mecanismo de lesión es:
Dedo en garra Fractura de cóndilo humeral
Luxación de codo
La crepitación palmar a la flexo-extensión indica disfunción Fractura supracondilea humeral
en la polea flexora. Fractura de tróclea

29.El auxiliar diagnóstico más útil en este caso es: La fractura supracondilea es la primera causa de lesiones
Radiografía simple traumáticas de codo en niños entre 5 y 8 años. El mecanismo
Ultrasonido en extensión de codo es el 95% de la causa de las fracturas
Electromiografía supracondileas.
Resonancia magnética cervical
32.¿En qué momento debe realizarse la exploración
El ultrasonido muestra la inflamación de las estructuras neurovascular completa de la extremidad?
blandas, además de permitir visualizar la polea y tendones de Inicial y al alta
manera dinámica visualizando la compresión por estenosis. Inicial, tras colocar inmovilización o reducción
Tras tratamiento quirúrgico
30.El tratamiento inicial deberá comprender lo Todas las anteriores
siguiente, excepto:
Reposo La exploración debe realizarse de manera inicial, tras
Inmovilización cualquier maniobra de reducción y colocación de
Liberación quirúrgica inmovilización y posterior al tratamiento quirúrgico sobre
AINES orales todo en presencia de edema importante por riesgo de
síndrome compartimental.
La liberación quirúrgica está indicada en caso de falla al
manejo conservador o de incapacidad para la extensión 33.En caso de neuropraxia cubital el paciente deberá
pasiva expresar hipoestesia en:
Dorso del primer dedo
Dorso del 1er, 2do y 3er dedo
Palma del 1er, 2do y 3er dedo
Dorso y palma de 4to y 5to dedo
El nervio cubital es un nervio mixto que provee la sensibilidad Radiografía simple de columna cervical
al dorso y palma del quinto dedo y la mitad cubital del Ultrasonido
cuarto. Electromiografía
Resonancia magnética de columna cervical

CASO CLÍNICO 13 El estudio recomendado para valoración de tenosinovitis de


Quervain es el ultrasonido, por ser un estudio dinámico para
Femenino de 29 años en puerperio de parto vaginal de 20 visualizar aumento de volumen en estructuras tendinosa y
días, que refiere una semana con dolor intenso en la base del estenosis de la corredera.
primer dedo de ambas manos que incrementa con la
actividad física, a la exploración con dolor en la base del 37.La maniobra de exploración que corrobora el
primer dedo en región dorsal irradiado a antebrazo. diagnóstico clínico es:
Túnel en mediano
34.El diagnóstico clínico de esta paciente será: Túnel en cubital
Túnel del carpo Phallen
Tenosinovitis de Quervain Filkenstein
Dedo en gatillo
Dedo en garra La maniobra de Finkelstein consiste en provocar el dolor de
forma selectiva al ejecutar un movimiento de desviación
La tenosinovitis de Quervain es una tendinitis que afecta a los cubital de la muñeca con el pulgar en flexión completa y el
tendones que ocupan el primer compartimento extensor de la resto de los dedos libres. Es positiva si provoca o aumenta de
mano, abductor largo y extensor corto del pulgar, forma significativa el dolor que sufre el paciente La maniobra
produciendo dolor con los movimientos del pulgar. El dolor de Filkenstein es patognomónica de Quervain.
por actividad física intensa o repetitiva del primer dedo en el
dorso sobre el trayecto de los tendones de la primera CASO CLÍNICO 14
corredera extensora es característico de esta entidad.
Femenino de 5 años que refiere dolor y limitación funcional
35.La etiología de este padecimiento es de índole: de codo derecho tras jugar con su padre a columpiarse con
Inflamatoria las manos, se visualiza edema leve pero el paciente no
Neoplásica permite la exploración por temor y dolor.
Autoinmune
Degenerativa 38.Ante los pocos datos clínicos de la exploración se
recomienda:
La tenosinovitis de Quervain es una entidad de origen Realizar radiografía AP y lateral
inflamatorio, a consecuencia del trauma repetitivo y Forzar al paciente a la exploración a pesar del dolor
sobreuso. Realizar ultrasonido de codo
Inmovilizar y revalorar posteriormente
36.El auxiliar diagnóstico más útil en este caso es:
Ante sospecha de lesión y en pacientes en quienes se Tomar proyecciones comparativas
encuentra dificultad para la exploración se recomienda Complementar con ultrasonido de codo
realizar radiografías en dos posiciones. Por la edad del Todas las anteriores
paciente y la presentación del cuadro, se sospecha de
subluxación de la cabeza del radio (codo de niñera), la cual En caso de sospecha diagnostica de lesión ósea en lesiones
produce dolor a la palpación sobre ella y mínimo edema de traumáticas de codo y ante duda en las proyecciones
estructuras circundantes, sin embargo, es recomendable convencionales se justifica la obtención de proyecciones
descartar mediante imagen la presencia de fracturas. oblicuas, comparativas o incluso ultrasonido o resonancia
magnética para tener certeza diagnóstica.
39.Tras la toma de radiografías el paciente regresa con
menos dolor y cooperador, la sospecha diagnostica
al momento será: CASO CLÍNICO 15
Esguince de codo
Contusión simple de codo Femenino de 17 años que realiza inversión forzada de tobillo
Paciente sano derecho al practicar atletismo en la escuela, a la exploración
Codo de niñera física con 1.56m 49 k, taquicardia y taquipnea, edema
perimaleolar lateral equimosis leve e incapacidad para el
Ante la sospecha clínica de subluxación de la cabeza del apoyo. Dolor perimaleolar y en tercio distal de peroné,
radio, se debe hacer maniobra de supinación y extensión que estabilidad no valorable.
no generan dolor. Habitualmente al realizarla reducen la
subluxación, ya sea realizada de manera activa o pasiva. Es 42.El principal factor de riesgo para el diagnóstico
probable que la mejoría se presente tras la supinación para la sospechado en esta paciente es:
toma de la radiografía AP. Edad
Peso
40.La complicación más frecuente de esta patología es: Actividad física
Recidiva Ninguno de los anteriores
Ninguna
Cubito valgo Las alteraciones anatómicas del pie, sobrepeso y actividades
Cubito varo laborales o deportivas son los principales factores de riesgo
para el desarrollo de los esguinces de tobillo.
La recidiva en la subluxación de la cabeza del radio (codo de
niñera) se presenta sobre todo en pacientes con hiperlaxitud 43.La toma de radiografías en este caso para corroborar
ligamentaria y con padres no educados que repiten el el diagnostico se basa en los criterios de:
mecanismo de lesión al realizar tracción del antebrazo. Katana
Watana
41.En caso de presentar duda diagnostica tras la Otawa
radiografía se recomienda: Kasawa
Tomar proyecciones oblicuas
Las reglas de Ottawa para tobillo sirven para valorar la cesárea por presentación pélvica, con Apgar 8-9, egresado de
necesidad de realizar radiografías para las lesiones de tobillo, su unidad de atención con la madre a las 48 horas.
ya que menos de un 15% de los pacientes con estas lesiones
tendrá una fractura significativa. Se requiere radiografía si 46.La revisión del recién nacido en busca de patología
hay dolor a la palpación en la parte posterior de los últimos 6 congénita de cadera se recomienda:
cm del peroné o de la tibia, si hay dolor a la palpación en la Al nacimiento, de manera sistemática
base del quinto o en la zona del hueso escafoides o A las 24 horas dirigida con maniobras especificas
incapacidad para ponerse de pie y soportar su peso o Después de dos semanas de nacimiento de manera
incapacidad para dar cuatro pasos en la exploración posterior. sistemática
Ninguna de las anteriores
44.El primer paso en la atención de este paciente
deberá corresponder a: El triage de patología de cadera y muchas otras se
Control de dolor recomienda de manera sistemática al nacimiento por
Colocación de vendaje antiedema personal calificado y con referencia al especialista en
Inmovilización con férula ortopedia en caso de duda diagnostica.
Preparar para quirófano
47.Las maniobras clínicas de evaluación de patología
El primer paso en la atención del paciente con un trauma congénita de cadera en el recién nacido son:
musculoesquelético agudo es el control del dolor que le Proporción de extremidades y asimetría de pliegues
brinde confort y facilite su exploración. Pistón y Galeazzi
Trendelemburg y limitación de rotación externa
45.La rehabilitación temprana del paciente con Barlow y ortolani
esguince de tobillo incluye las siguientes medidas
excepto: Ortolani (cadera luxada) y Barlow (cadera luxable) son las
Calor local y masaje suave en la zona afectada maniobras de exploración positivas de mayor utilidad en el
Evitar apoyo en las primeras 48 horas diagnóstico de DDC hasta los 3 meses de edad.
Movilizar activa de tobillo a tolerancia Posteriormente y debido al acortamiento y contracturas son
Vendaje elástico no compresivo útiles las maniobras de pistón y Galeazzi y al iniciar la
marcha, Trendelemburg. La asimetría de pliegues no es una
Todas las medidas mencionadas son adecuadas para el manejo maniobra.
de esguince de tobillo, excepto la aplicación de calor, de
hecho, en trauma agudo se indica la colocación de hielo por CASO CLÍNICO 17
20 minutos cada 8 horas.
A 13 year old boy has had thigh pain for the past several
CASO CLÍNICO 16 months. He denies any history of trauma. Examination
reveals a large deeply fixed, soft-tissue mass in the thigh.
Recién nacido masculino de 38 semanas de gestación, Laboratory results show an elevated erythrocyte
producto de primera gesta de madre sana, obtenido por sedimentation rate and leukocytosis.
48.Which is the next step in this patient’s care?
Next level attention center refer
Take plain x-ray of the tight
Prescribe NAIDs and observe
All of the above

Although sending the patient to the next level of attention is


a correct way to proceed, it is recommended to take images
to complement diagnosis to reassure the presumptive
diagnosis.

49.A plain radiograph shows a lytic aggressive lesion


within the diaphysis of the femur with periostic
reaction accompanied by a large soft tissue mass.
The most likely diagnosis is:
Ewing sarcoma
Giant cell tumor
Osteosarcoma
Chondrosarcoma

Based on the characteristics and place of the lesion, the most


likely diagnosis is osteosarcoma. Giant cell tumor and
chondrosarcoma have metaphyseal location and
chondrosarcoma typically is found in older age groups.

50.Which of the following factors is associated with the


worst prognosis in sarcomas?
Size greater than 15cm
Extra-compartmental involvement
Number of mitotic figures per high-power field (grade)
Presence of metastases

Although factors such as a high-grade tumor and large size


are associated with decreased survival, the presence of
metastases carries the worst prognosis.

También podría gustarte